Clinical utility of warfarin pharmacogenomics

Article Type
Changed
Mon, 10/02/2017 - 15:09
Display Headline
Clinical utility of warfarin pharmacogenomics

To the Editor: We previously addressed whether VKORC1 and CYP2C9 pharmacogenomic testing should be considered when prescribing warfarin.1 Our recommendation, based on available evidence at that time, was that physicians should consider pharmacogenomic testing for any patient who is started on warfarin therapy.

Since the publication of this recommendation, two major trials, COAG (Clarification of Optimal Anticoagulation Through Genetics)2 and EU-PACT (European Pharmacogenetics of Anticoagulant Therapy-Warfarin),3 were published along with commentaries debating the clinical utility of warfarin pharmacogenomics.4–15 Based on these publications, we would like to update our recommendations for pharmacogenomic testing for warfarin therapy.

COAG compared the efficacy of a clinical algorithm or a clinical algorithm plus VKORC1 and CYP2C9 genotyping to guide warfarin dosage. At the end of 4 weeks, the mean percentage of time within the therapeutic international normalized ratio (INR) range was 45.4% for those in the clinical algorithm arm and 45.2% for those in the genotyping arm (95% confidence interval [CI] –3.4 to 3.1, P = .91). For both treatment groups, clinical data that included body surface area, age, target INR, concomitantly prescribed drugs, and smoking status were used to predict warfarin dose, with the genotyping arm including VKORC1 and CYP2C9. Although VKORC1 and CYP2C9 genotyping offered no additional benefit, caution should be used when extrapolating this conclusion to clinical settings in which warfarin therapy is initiated using a standardized starting dose (eg, 5 mg daily) instead of a clinical dosing algorithm.

Of interest, in the COAG trial, among black patients, the mean percentage of time in the therapeutic INR range was significantly less for those in the genotype-guided arm than for those in the clinically guided arm—ie, 35.2% vs 43.5% (95% CI –15.0 to –2.0, P = .01). The percentage of time with therapeutic INR has been identified as a surrogate marker for poor outcomes such as death, stroke, or major hemorrhage, with those with a lower percentage of time in therapeutic INR being at greater risk of an adverse event.16 Wan et al17 demonstrated that a 6.9% improvement of time in therapeutic INR decreased the risk of major hemorrhage by one event per 100 patient-years.17 Therefore, black patients in the COAG genotyping arm may have been at greater risk for an adverse event because of a lower observed percentage of time within the therapeutic INR range.

In the COAG trial, genotyping was done for only one VKORC1 variant and for two CYP2C9 alleles (CYP2C9*2, and CYP2C9*3). Other genetic variants are of clinical importance for warfarin dosing in black patients, and the lack of genotyping for these additional variants may explain why black patients in the genotyping arm performed worse.5,7,11 In particular, CYP2C9*8 may be an important predictor of warfarin dose in black patients.18

EU-PACT compared the efficacy of standardized warfarin dosing and that of a clinical algorithm.3 Patients in the standardized dosing arm were prescribed warfarin 10 mg on the first day of treatment (5 mg for those over age 75), and 5 mg on days 2 and 3, with subsequent dosing adjustments based on INR. Patients in the genotyping arm were prescribed warfarin based on an algorithm that incorporated clinical data that included body surface area, age, and concomitantly prescribed drugs, as well as VKORC1 and CYP2C9 genotypes. At the end of 12 weeks, the mean percentage of time in the therapeutic INR range was 60.3% for those in the standardized-dosing arm and 67.4% for those in the genotyping arm (95% CI 3.3 to 10.6, P < .001).2 The approximate 7% improvement in percentage of time in the therapeutic INR range may predict a lower risk of hemorrhage for those in the genotyping arm.17 Although patients in the genotyping arm had a higher percentage of time in the therapeutic INR range, it is unclear whether genotyping alone is superior to standardized dosing because the dosing algorithm used both clinical data and genotype data.

There are substantial differences between the COAG and EU-PACT trials, including dosing schemes, racial diversity, and trial length, and these differences could have contributed to the conflicting results. Based on these two trials, a possible conclusion is that genotype-guided warfarin dosing may be superior to standardized dosing, but may be no better than utilizing a clinical algorithm in white patients. For black patients, additional studies are needed to determine which genetic variants are of importance for guiding warfarin dosing.

We would like to update the recommendations we made in our previously published article,1 to state that genotyping for CYP2C9 and VKORC1 may be of clinical utility in white patients depending on whether standardized dosing or a clinical algorithm is used to initiate warfarin therapy. Routine genotyping in black patients is not recommended until further studies clarify which genetic variants are of importance for guiding warfarin dosing.

The ongoing Genetics Informatics Trial of Warfarin to Prevent Venous Thrombosis may bring much needed clarity to the clinical utility of warfarin pharmacogenomics. We hope to publish a more detailed update of our 2013 article after completion of that trial.

References
  1. Rouse M, Cristiani C, Teng KA. Should we use pharmacogenetic testing when prescribing warfarin? Cleve Clin J Med 2013; 80:483–486.
  2. Kimmel SE, French B, Kasner SE, et al; COAG Investigators. A pharmacogenetic versus a clinical algorithm for warfarin dosing. N Engl J Med 2013; 369:2283–2293.
  3. Pirmohamed M, Burnside G, Eriksson N, et al; EU-PACT Group. A randomized trial of genotype-guided dosing of warfarin. N Engl J Med 2013; 369:2294–2303.
  4. Cavallari LH, Kittles RA, Perera MA. Genotype-guided dosing of vitamin K antagonists. N Engl J Med 2014; 370:1763.
  5. Cavallari LH, Nutescu EA. Warfarin pharmacogenetics: to genotype or not to genotype, that is the question. Clin Pharmacol Ther 2014; 96:22–24.
  6. Daneshjou R, Klein TE, Altman RB. Genotype-guided dosing of vitamin K antagonists. N Engl J Med 2014; 370:1762–1763.
  7. Hernandez W, Gamazon ER, Aquino-Michaels K, et al. Ethnicity-specific pharmacogenetics: the case of warfarin in African Americans. Pharmacogenomics J 2014; 14:223–228.
  8. Kimmel SE, French B, Geller NL; COAG Investigators. Genotype-guided dosing of vitamin K antagonists. N Engl J Med 2014; 370:1763–1764.
  9. Koller EA, Roche JC, Rollins JA. Genotype-guided dosing of vitamin K antagonists. N Engl J Med 2014; 370:1761.
  10. Pereira NL, Rihal CS, Weinshilboum RM. Genotype-guided dosing of vitamin K antagonists. N Engl J Med 2014; 370:1762.
  11. Perera MA, Cavallari LH, Johnson JA. Warfarin pharmacogenetics: an illustration of the importance of studies in minority populations. Clin Pharmacol Ther 2014; 95:242–244.
  12. Pirmohamed M, Wadelius M, Kamali F; EU-PACT Group. Genotype-guided dosing of vitamin K antagonists. N Engl J Med 2014; 370:1764–1765.
  13. Schwarz UI, Kim RB, Tirona RG. Genotype-guided dosing of vitamin K antagonists. N Engl J Med 2014; 370:1761–1762.
  14. Scott SA, Lubitz SA. Warfarin pharmacogenetic trials: is there a future for pharmacogenetic-guided dosing? Pharmacogenomics 2014; 15:719–722.
  15. Zineh I, Pacanowski M, Woodcock J. Pharmacogenetics and coumarin dosing—recalibrating expectations. N Engl J Med 2013; 369:2273–2275.
  16. Hylek EM. Vitamin K antagonists and time in the therapeutic range: implications, challenges, and strategies for improvement. J Thromb Thrombolysis 2013; 35:333–335.
  17. Wan Y, Heneghan C, Perera R, et al. Anticoagulation control and prediction of adverse events in patients with atrial fibrillation: a systematic review. Circ Cardiovasc Qual Outcomes 2008;1:84-91.
  18. Nagai R, Ohara M, Cavallari LH, et al. Factors influencing pharmacokinetics of warfarin in African-Americans: implications for pharmacogenetic dosing algorithms. Pharmacogenomics 2015;16:217–225.
Article PDF
Author and Disclosure Information

D. Max Smith, BSPS
Department of Pharmacy, Cleveland Clinic

Cari Cristiani, PharmD, BCPS, BCACP
Department of Pharmacy, Cleveland Clinic

Kathryn A. Teng, MD, FACP
Director, Internal Medicine and Community Medicine, MetroHealth System, Cleveland, OH

J. Kevin Hicks, PharmD, PhD
Department of Pharmacy, Genomic Medicine Institute, Cleveland Clinic

Issue
Cleveland Clinic Journal of Medicine - 82(5)
Publications
Topics
Page Number
268-269
Legacy Keywords
warfarin, pharmacogenomics, CYP2C9, VKORC1, Max Smith, Cari Cristiani, Kathryn Teng, Kevin Hicks
Sections
Author and Disclosure Information

D. Max Smith, BSPS
Department of Pharmacy, Cleveland Clinic

Cari Cristiani, PharmD, BCPS, BCACP
Department of Pharmacy, Cleveland Clinic

Kathryn A. Teng, MD, FACP
Director, Internal Medicine and Community Medicine, MetroHealth System, Cleveland, OH

J. Kevin Hicks, PharmD, PhD
Department of Pharmacy, Genomic Medicine Institute, Cleveland Clinic

Author and Disclosure Information

D. Max Smith, BSPS
Department of Pharmacy, Cleveland Clinic

Cari Cristiani, PharmD, BCPS, BCACP
Department of Pharmacy, Cleveland Clinic

Kathryn A. Teng, MD, FACP
Director, Internal Medicine and Community Medicine, MetroHealth System, Cleveland, OH

J. Kevin Hicks, PharmD, PhD
Department of Pharmacy, Genomic Medicine Institute, Cleveland Clinic

Article PDF
Article PDF
Related Articles

To the Editor: We previously addressed whether VKORC1 and CYP2C9 pharmacogenomic testing should be considered when prescribing warfarin.1 Our recommendation, based on available evidence at that time, was that physicians should consider pharmacogenomic testing for any patient who is started on warfarin therapy.

Since the publication of this recommendation, two major trials, COAG (Clarification of Optimal Anticoagulation Through Genetics)2 and EU-PACT (European Pharmacogenetics of Anticoagulant Therapy-Warfarin),3 were published along with commentaries debating the clinical utility of warfarin pharmacogenomics.4–15 Based on these publications, we would like to update our recommendations for pharmacogenomic testing for warfarin therapy.

COAG compared the efficacy of a clinical algorithm or a clinical algorithm plus VKORC1 and CYP2C9 genotyping to guide warfarin dosage. At the end of 4 weeks, the mean percentage of time within the therapeutic international normalized ratio (INR) range was 45.4% for those in the clinical algorithm arm and 45.2% for those in the genotyping arm (95% confidence interval [CI] –3.4 to 3.1, P = .91). For both treatment groups, clinical data that included body surface area, age, target INR, concomitantly prescribed drugs, and smoking status were used to predict warfarin dose, with the genotyping arm including VKORC1 and CYP2C9. Although VKORC1 and CYP2C9 genotyping offered no additional benefit, caution should be used when extrapolating this conclusion to clinical settings in which warfarin therapy is initiated using a standardized starting dose (eg, 5 mg daily) instead of a clinical dosing algorithm.

Of interest, in the COAG trial, among black patients, the mean percentage of time in the therapeutic INR range was significantly less for those in the genotype-guided arm than for those in the clinically guided arm—ie, 35.2% vs 43.5% (95% CI –15.0 to –2.0, P = .01). The percentage of time with therapeutic INR has been identified as a surrogate marker for poor outcomes such as death, stroke, or major hemorrhage, with those with a lower percentage of time in therapeutic INR being at greater risk of an adverse event.16 Wan et al17 demonstrated that a 6.9% improvement of time in therapeutic INR decreased the risk of major hemorrhage by one event per 100 patient-years.17 Therefore, black patients in the COAG genotyping arm may have been at greater risk for an adverse event because of a lower observed percentage of time within the therapeutic INR range.

In the COAG trial, genotyping was done for only one VKORC1 variant and for two CYP2C9 alleles (CYP2C9*2, and CYP2C9*3). Other genetic variants are of clinical importance for warfarin dosing in black patients, and the lack of genotyping for these additional variants may explain why black patients in the genotyping arm performed worse.5,7,11 In particular, CYP2C9*8 may be an important predictor of warfarin dose in black patients.18

EU-PACT compared the efficacy of standardized warfarin dosing and that of a clinical algorithm.3 Patients in the standardized dosing arm were prescribed warfarin 10 mg on the first day of treatment (5 mg for those over age 75), and 5 mg on days 2 and 3, with subsequent dosing adjustments based on INR. Patients in the genotyping arm were prescribed warfarin based on an algorithm that incorporated clinical data that included body surface area, age, and concomitantly prescribed drugs, as well as VKORC1 and CYP2C9 genotypes. At the end of 12 weeks, the mean percentage of time in the therapeutic INR range was 60.3% for those in the standardized-dosing arm and 67.4% for those in the genotyping arm (95% CI 3.3 to 10.6, P < .001).2 The approximate 7% improvement in percentage of time in the therapeutic INR range may predict a lower risk of hemorrhage for those in the genotyping arm.17 Although patients in the genotyping arm had a higher percentage of time in the therapeutic INR range, it is unclear whether genotyping alone is superior to standardized dosing because the dosing algorithm used both clinical data and genotype data.

There are substantial differences between the COAG and EU-PACT trials, including dosing schemes, racial diversity, and trial length, and these differences could have contributed to the conflicting results. Based on these two trials, a possible conclusion is that genotype-guided warfarin dosing may be superior to standardized dosing, but may be no better than utilizing a clinical algorithm in white patients. For black patients, additional studies are needed to determine which genetic variants are of importance for guiding warfarin dosing.

We would like to update the recommendations we made in our previously published article,1 to state that genotyping for CYP2C9 and VKORC1 may be of clinical utility in white patients depending on whether standardized dosing or a clinical algorithm is used to initiate warfarin therapy. Routine genotyping in black patients is not recommended until further studies clarify which genetic variants are of importance for guiding warfarin dosing.

The ongoing Genetics Informatics Trial of Warfarin to Prevent Venous Thrombosis may bring much needed clarity to the clinical utility of warfarin pharmacogenomics. We hope to publish a more detailed update of our 2013 article after completion of that trial.

To the Editor: We previously addressed whether VKORC1 and CYP2C9 pharmacogenomic testing should be considered when prescribing warfarin.1 Our recommendation, based on available evidence at that time, was that physicians should consider pharmacogenomic testing for any patient who is started on warfarin therapy.

Since the publication of this recommendation, two major trials, COAG (Clarification of Optimal Anticoagulation Through Genetics)2 and EU-PACT (European Pharmacogenetics of Anticoagulant Therapy-Warfarin),3 were published along with commentaries debating the clinical utility of warfarin pharmacogenomics.4–15 Based on these publications, we would like to update our recommendations for pharmacogenomic testing for warfarin therapy.

COAG compared the efficacy of a clinical algorithm or a clinical algorithm plus VKORC1 and CYP2C9 genotyping to guide warfarin dosage. At the end of 4 weeks, the mean percentage of time within the therapeutic international normalized ratio (INR) range was 45.4% for those in the clinical algorithm arm and 45.2% for those in the genotyping arm (95% confidence interval [CI] –3.4 to 3.1, P = .91). For both treatment groups, clinical data that included body surface area, age, target INR, concomitantly prescribed drugs, and smoking status were used to predict warfarin dose, with the genotyping arm including VKORC1 and CYP2C9. Although VKORC1 and CYP2C9 genotyping offered no additional benefit, caution should be used when extrapolating this conclusion to clinical settings in which warfarin therapy is initiated using a standardized starting dose (eg, 5 mg daily) instead of a clinical dosing algorithm.

Of interest, in the COAG trial, among black patients, the mean percentage of time in the therapeutic INR range was significantly less for those in the genotype-guided arm than for those in the clinically guided arm—ie, 35.2% vs 43.5% (95% CI –15.0 to –2.0, P = .01). The percentage of time with therapeutic INR has been identified as a surrogate marker for poor outcomes such as death, stroke, or major hemorrhage, with those with a lower percentage of time in therapeutic INR being at greater risk of an adverse event.16 Wan et al17 demonstrated that a 6.9% improvement of time in therapeutic INR decreased the risk of major hemorrhage by one event per 100 patient-years.17 Therefore, black patients in the COAG genotyping arm may have been at greater risk for an adverse event because of a lower observed percentage of time within the therapeutic INR range.

In the COAG trial, genotyping was done for only one VKORC1 variant and for two CYP2C9 alleles (CYP2C9*2, and CYP2C9*3). Other genetic variants are of clinical importance for warfarin dosing in black patients, and the lack of genotyping for these additional variants may explain why black patients in the genotyping arm performed worse.5,7,11 In particular, CYP2C9*8 may be an important predictor of warfarin dose in black patients.18

EU-PACT compared the efficacy of standardized warfarin dosing and that of a clinical algorithm.3 Patients in the standardized dosing arm were prescribed warfarin 10 mg on the first day of treatment (5 mg for those over age 75), and 5 mg on days 2 and 3, with subsequent dosing adjustments based on INR. Patients in the genotyping arm were prescribed warfarin based on an algorithm that incorporated clinical data that included body surface area, age, and concomitantly prescribed drugs, as well as VKORC1 and CYP2C9 genotypes. At the end of 12 weeks, the mean percentage of time in the therapeutic INR range was 60.3% for those in the standardized-dosing arm and 67.4% for those in the genotyping arm (95% CI 3.3 to 10.6, P < .001).2 The approximate 7% improvement in percentage of time in the therapeutic INR range may predict a lower risk of hemorrhage for those in the genotyping arm.17 Although patients in the genotyping arm had a higher percentage of time in the therapeutic INR range, it is unclear whether genotyping alone is superior to standardized dosing because the dosing algorithm used both clinical data and genotype data.

There are substantial differences between the COAG and EU-PACT trials, including dosing schemes, racial diversity, and trial length, and these differences could have contributed to the conflicting results. Based on these two trials, a possible conclusion is that genotype-guided warfarin dosing may be superior to standardized dosing, but may be no better than utilizing a clinical algorithm in white patients. For black patients, additional studies are needed to determine which genetic variants are of importance for guiding warfarin dosing.

We would like to update the recommendations we made in our previously published article,1 to state that genotyping for CYP2C9 and VKORC1 may be of clinical utility in white patients depending on whether standardized dosing or a clinical algorithm is used to initiate warfarin therapy. Routine genotyping in black patients is not recommended until further studies clarify which genetic variants are of importance for guiding warfarin dosing.

The ongoing Genetics Informatics Trial of Warfarin to Prevent Venous Thrombosis may bring much needed clarity to the clinical utility of warfarin pharmacogenomics. We hope to publish a more detailed update of our 2013 article after completion of that trial.

References
  1. Rouse M, Cristiani C, Teng KA. Should we use pharmacogenetic testing when prescribing warfarin? Cleve Clin J Med 2013; 80:483–486.
  2. Kimmel SE, French B, Kasner SE, et al; COAG Investigators. A pharmacogenetic versus a clinical algorithm for warfarin dosing. N Engl J Med 2013; 369:2283–2293.
  3. Pirmohamed M, Burnside G, Eriksson N, et al; EU-PACT Group. A randomized trial of genotype-guided dosing of warfarin. N Engl J Med 2013; 369:2294–2303.
  4. Cavallari LH, Kittles RA, Perera MA. Genotype-guided dosing of vitamin K antagonists. N Engl J Med 2014; 370:1763.
  5. Cavallari LH, Nutescu EA. Warfarin pharmacogenetics: to genotype or not to genotype, that is the question. Clin Pharmacol Ther 2014; 96:22–24.
  6. Daneshjou R, Klein TE, Altman RB. Genotype-guided dosing of vitamin K antagonists. N Engl J Med 2014; 370:1762–1763.
  7. Hernandez W, Gamazon ER, Aquino-Michaels K, et al. Ethnicity-specific pharmacogenetics: the case of warfarin in African Americans. Pharmacogenomics J 2014; 14:223–228.
  8. Kimmel SE, French B, Geller NL; COAG Investigators. Genotype-guided dosing of vitamin K antagonists. N Engl J Med 2014; 370:1763–1764.
  9. Koller EA, Roche JC, Rollins JA. Genotype-guided dosing of vitamin K antagonists. N Engl J Med 2014; 370:1761.
  10. Pereira NL, Rihal CS, Weinshilboum RM. Genotype-guided dosing of vitamin K antagonists. N Engl J Med 2014; 370:1762.
  11. Perera MA, Cavallari LH, Johnson JA. Warfarin pharmacogenetics: an illustration of the importance of studies in minority populations. Clin Pharmacol Ther 2014; 95:242–244.
  12. Pirmohamed M, Wadelius M, Kamali F; EU-PACT Group. Genotype-guided dosing of vitamin K antagonists. N Engl J Med 2014; 370:1764–1765.
  13. Schwarz UI, Kim RB, Tirona RG. Genotype-guided dosing of vitamin K antagonists. N Engl J Med 2014; 370:1761–1762.
  14. Scott SA, Lubitz SA. Warfarin pharmacogenetic trials: is there a future for pharmacogenetic-guided dosing? Pharmacogenomics 2014; 15:719–722.
  15. Zineh I, Pacanowski M, Woodcock J. Pharmacogenetics and coumarin dosing—recalibrating expectations. N Engl J Med 2013; 369:2273–2275.
  16. Hylek EM. Vitamin K antagonists and time in the therapeutic range: implications, challenges, and strategies for improvement. J Thromb Thrombolysis 2013; 35:333–335.
  17. Wan Y, Heneghan C, Perera R, et al. Anticoagulation control and prediction of adverse events in patients with atrial fibrillation: a systematic review. Circ Cardiovasc Qual Outcomes 2008;1:84-91.
  18. Nagai R, Ohara M, Cavallari LH, et al. Factors influencing pharmacokinetics of warfarin in African-Americans: implications for pharmacogenetic dosing algorithms. Pharmacogenomics 2015;16:217–225.
References
  1. Rouse M, Cristiani C, Teng KA. Should we use pharmacogenetic testing when prescribing warfarin? Cleve Clin J Med 2013; 80:483–486.
  2. Kimmel SE, French B, Kasner SE, et al; COAG Investigators. A pharmacogenetic versus a clinical algorithm for warfarin dosing. N Engl J Med 2013; 369:2283–2293.
  3. Pirmohamed M, Burnside G, Eriksson N, et al; EU-PACT Group. A randomized trial of genotype-guided dosing of warfarin. N Engl J Med 2013; 369:2294–2303.
  4. Cavallari LH, Kittles RA, Perera MA. Genotype-guided dosing of vitamin K antagonists. N Engl J Med 2014; 370:1763.
  5. Cavallari LH, Nutescu EA. Warfarin pharmacogenetics: to genotype or not to genotype, that is the question. Clin Pharmacol Ther 2014; 96:22–24.
  6. Daneshjou R, Klein TE, Altman RB. Genotype-guided dosing of vitamin K antagonists. N Engl J Med 2014; 370:1762–1763.
  7. Hernandez W, Gamazon ER, Aquino-Michaels K, et al. Ethnicity-specific pharmacogenetics: the case of warfarin in African Americans. Pharmacogenomics J 2014; 14:223–228.
  8. Kimmel SE, French B, Geller NL; COAG Investigators. Genotype-guided dosing of vitamin K antagonists. N Engl J Med 2014; 370:1763–1764.
  9. Koller EA, Roche JC, Rollins JA. Genotype-guided dosing of vitamin K antagonists. N Engl J Med 2014; 370:1761.
  10. Pereira NL, Rihal CS, Weinshilboum RM. Genotype-guided dosing of vitamin K antagonists. N Engl J Med 2014; 370:1762.
  11. Perera MA, Cavallari LH, Johnson JA. Warfarin pharmacogenetics: an illustration of the importance of studies in minority populations. Clin Pharmacol Ther 2014; 95:242–244.
  12. Pirmohamed M, Wadelius M, Kamali F; EU-PACT Group. Genotype-guided dosing of vitamin K antagonists. N Engl J Med 2014; 370:1764–1765.
  13. Schwarz UI, Kim RB, Tirona RG. Genotype-guided dosing of vitamin K antagonists. N Engl J Med 2014; 370:1761–1762.
  14. Scott SA, Lubitz SA. Warfarin pharmacogenetic trials: is there a future for pharmacogenetic-guided dosing? Pharmacogenomics 2014; 15:719–722.
  15. Zineh I, Pacanowski M, Woodcock J. Pharmacogenetics and coumarin dosing—recalibrating expectations. N Engl J Med 2013; 369:2273–2275.
  16. Hylek EM. Vitamin K antagonists and time in the therapeutic range: implications, challenges, and strategies for improvement. J Thromb Thrombolysis 2013; 35:333–335.
  17. Wan Y, Heneghan C, Perera R, et al. Anticoagulation control and prediction of adverse events in patients with atrial fibrillation: a systematic review. Circ Cardiovasc Qual Outcomes 2008;1:84-91.
  18. Nagai R, Ohara M, Cavallari LH, et al. Factors influencing pharmacokinetics of warfarin in African-Americans: implications for pharmacogenetic dosing algorithms. Pharmacogenomics 2015;16:217–225.
Issue
Cleveland Clinic Journal of Medicine - 82(5)
Issue
Cleveland Clinic Journal of Medicine - 82(5)
Page Number
268-269
Page Number
268-269
Publications
Publications
Topics
Article Type
Display Headline
Clinical utility of warfarin pharmacogenomics
Display Headline
Clinical utility of warfarin pharmacogenomics
Legacy Keywords
warfarin, pharmacogenomics, CYP2C9, VKORC1, Max Smith, Cari Cristiani, Kathryn Teng, Kevin Hicks
Legacy Keywords
warfarin, pharmacogenomics, CYP2C9, VKORC1, Max Smith, Cari Cristiani, Kathryn Teng, Kevin Hicks
Sections
Disallow All Ads
Alternative CME
Article PDF Media

Should we use pharmacogenetic testing when prescribing warfarin?

Article Type
Changed
Mon, 10/02/2017 - 15:10
Display Headline
Should we use pharmacogenetic testing when prescribing warfarin?

The answer is not clear. There is evidence in favor of pharmacogenetic testing, but not yet enough to strongly recommend it. However, we do believe that physicians should consider it when starting patients on warfarin therapy.

See related commentary

WARFARIN HAS A NARROW THERAPEUTIC WINDOW

Although newer drugs are available, warfarin is still the most commonly used oral anticoagulant for preventing and treating thromboembolism.1 It is highly effective but has a narrow therapeutic window and wide interindividual variability in dosage requirements, which poses challenges to achieving adequate anticoagulation.1–3 Inappropriate dosing contributes to a high rate of bleeding events and emergency room visits.4

Warfarin is monitored using the prothrombin time. Because the prothrombin time varies depending on the assay used, the standardized value called the international normalized ratio (INR) is more commonly used.

Clinical factors such as age, body size, and drug interactions affect warfarin dosage requirements and are important to consider,5 even though they account for only 15% to 20% of the variability in warfarin dose.6

Genetic factors also affect warfarin dosage requirements. The combination of genetic and clinical factors accounts for up to 47% of the dose variability.7

GENES THAT AFFECT WARFARIN

Several genes are known to influence warfarin’s pharmacokinetics and pharmacodynamics. Of these, the two most clinically relevant and well studied are CYP2C9 (which codes for cytochrome P450 2C9) and VKORC1 (which codes for vitamin K epoxide reductase).7 These genes are polymorphic, with some variants producing less-active enzymes that allow warfarin to be more active. Therefore, patients who carry these variants need lower doses of this drug (see below).

CYP2C9 variants

The CYP2C9 gene has several variants. Of these, CYP2C9*2 and CYP2C9*3 are associated with the lowest enzyme activity.

Patients with either of these variants require significantly lower warfarin doses to reach therapeutic levels than those with the wild-type gene (ie, CYP2C9*1). CYP2C9*2 reduces warfarin clearance by 40%, and the CYP2C9*3 variant reduces it by 75%.7 Having a *2 or *3 allele increases the risk of bleeding during warfarin therapy and the time needed to achieve a stable dose.8 Other variants associated with lower warfarin dose requirements are *5, *6, and *11.

The prevalence of these variants is significantly higher in people of European ancestry (roughly one-third) than in Asian people and African Americans,7 although no one has recommended not testing in these low-prevalence populations. Limdi et al9 reported that by including the *5, *6, and *11 variants in genetic testing (in addition to *2 and *3), they could identify more African Americans (9.7%) who carried at least one of these abnormal variants than reported previously. Differences among ethnic groups need to be taken into account when interpreting pharmacogenetic studies.

VKORC1 variants

Patients also need lower doses of warfarin if they carry the VKORC1 −1639G>A variant, and they spend more time with an INR above the therapeutic range and have higher overall INR values. However, having this variant does not appear to increase the risk of bleeding.

The −1639G>A variant is the most common variant of VKORC1. Rarer ones have also been described, but most commercially available tests do not detect them.

Racial differences exist in the prevalence rates of the various VKORC1 polymorphisms, with the most sensitive (low-dose) genotype predominating in Asians and the least sensitive (high-dose) genotype predominating in African Americans. Over 50% of people of European ancestry carry the intermediate-sensitivity genotype (typical dose).7

CURRENT RECOMMENDATIONS FOR OR AGAINST TESTING

FDA labeling

In 2007, the US Food and Drug Administration (FDA) required that the warfarin package insert carry information about initial dosing based on CYP2C9 and VKORC1 testing. This recommendation was revised in 2010 to include a table to help clinicians select an initial warfarin dose if CYP2C9 and VKORC1 genotype information is available. However, the FDA does not require pharmacogenetic testing, leaving the decision to the discretion of the clinician.7

American College of Chest Physicians

The American College of Chest Physicians recommends against routine pharmacogenetic testing (grade 1B) because of a lack of evidence that it improves clinical end points or that it is cost-effective.5

WHAT EVIDENCE SUPORTS GENETIC TESTING TO GUIDE WARFARIN THERAPY?

To date, no large randomized, controlled trial has been published that looked at clinical outcomes with warfarin dosing based on pharmacogenetic testing. However, several smaller studies have suggested it is beneficial.

One trial found that when dosing was informed by pharmacogenetic testing, patients had significantly more time in the therapeutic range, a lower percentage of INRs greater than 4 or less than 1.5, and fewer serious adverse events (death, myocardial infarction, stroke, thromboembolism, and clinically significant bleeding events).10 Patients whose dosage was determined using pharmacogenetic algorithms as opposed to traditional clinical algorithms maintained a therapeutic INR more consistently.11

In addition, compared with historical controls, patients whose physician used pharmacogenetic testing to guide warfarin dosing had a rate of hospitalization 31% lower and a rate of hospitalization specifically for bleeding or thromboembolism 28% lower during 6 months of follow-up.12,13

Several studies have attempted to assess the cost-effectiveness and utility of pharmacogenetic testing in warfarin therapy. As yet, the results have been inconclusive.14 Larger prospective trials are under way and are estimated to be completed in late 2013.15 These include:

  • COAG (Clarification of Optimal Anticoagulation Through Genetics)
  • GIFT (Genetics Informatics Trial of Warfarin to Prevent Venous Thrombosis)
  • EU-PACT (European Pharmacogenetics of Anticoagulant Therapy-Warfarin).

We hope these studies will provide greater clarity on the clinical utility and cost-effectiveness of pharmacogenetic testing to guide warfarin dosing.

 

 

HOW SHOULD GENETIC INFORMATION BE USED TO GUIDE OR ALTER THERAPY?

Algorithms are available for estimating initial and maintenance warfarin doses based on genetic information (CYP2C9 and VKORC1), race or ethnicity, age, sex, body mass index, smoking status, and other medications taken. In addition, models incorporating the INR on day 4 and days 6 to 11 have been developed for dose refinement.15 The algorithms explain 30% to 60% of the variability of the data, with lower values for African Americans.7

A well-developed dosing model that includes traditional clinical factors and patient genetic status is publicly available online at www.warfarindosing.org.4

CPIC: A leader in applied pharmacogenetics

In late 2009, PharmGKB joined forces with the Pharmacogenomics Research Network of the National Institutes of Health to form the Clinical Pharmacogenetics Implementation Consortium (CPIC). This organization issues guidelines that are written by expert clinicians and scientists and then are peer-reviewed, published in leading journals, and simultaneously posted to the PharmGKB website along with supplemental information and updates.

CPIC’s goal is to review the current evidence and to address barriers to the adoption of pharmacogenetic testing into clinical practice. Its guidelines do not advise when or which pharmacogenetic tests should be ordered. Rather, they provide guidance on interpreting and applying such testing, should the test results be available.7

CPIC has guidelines on CYP2C9 and VKORC1 genotypes and warfarin dosing.8 If a patient’s genetic information is available, CPIC strongly recommends the use of pharmacogenetic algorithm-based dosing. If such an algorithm is not accessible, use of a genotype dosing table is recommended.8

Monitoring is still needed

Many factors can affect an individual’s response to warfarin above and beyond the above-noted clinical and genetic traits. These include diet, concomitant medications (both prescription and over-the-counter and herbal), and disease state. There may also be additional genetic polymorphisms not yet identified in various racial and ethnic groups that may affect dosing requirements. And as with all medications, patient compliance and dosing errors have a large potential to affect individual response. Therefore, clinicians should still be diligent about clinical monitoring.15

Most useful for initial dose

As with most pharmacogenetic information, the greatest benefit can be achieved when this information is used to guide the initial dose, although there is also some effect noted when this information is known and acted upon into the 2nd week of treatment.8

Patients on long-term warfarin treatment with stable doses and those unable to achieve stable dosing because of variable adherence or dietary vitamin K intake are less likely to benefit from genetic testing.

There are no published guidelines on the utility of pharmacogenetic testing if a patient is already on a stable dose of warfarin or has a known historical stable dose. There are also no published guidelines on changing the frequency of monitoring based on known genotype.

In children, the data are sparse at this time regarding the utility of pharmacogenetically informed dosing.

HOW DOES ONE ORDER TESTING, AND WHAT IS THE COST?

The FDA has approved four warfarin pharmacogenetic test kits. To be used in clinical decision-making, these tests must be done in a laboratory certified by the Clinical Laboratory Improvement Amendments (CLIA) program.

Testing typically costs a few hundred dollars and may take days for results to be returned if not available on site.15 At Cleveland Clinic, CYP2C9 and VKORC1 testing can be run in-house at a cost of about $700. Generally, many third-party payers do not reimburse for testing without a prior-approval process.

TO TEST OR NOT TO TEST

Pharmacogenetic testing is available and may help optimize warfarin dosing early in treatment, as well as help maintain therapeutic INRs more consistently. There is preliminary evidence that using this information to guide dosing improves clinical outcomes. Several large trials are under way to address additional questions of clinical utility, with results expected in the next year. There are also readily available decision-support tools to guide therapeutic dosing, and when pharmacogenetic test results are available, utilization of a warfarin dosing algorithm is recommended.

The largest barrier remaining appears to be cost (relative to perceived benefit), and until larger trials of clinical utility and cost-effectiveness are completed and analyzed, hurdles exist to obtaining coverage for such testing.

If it is readily available (and can be paid for by insurance companies or out-of-pocket) and test results can be obtained within 24 to 48 hours or before prescribing, pharmacogenetic testing can be a valuable tool to guide and manage warfarin dosing. Particularly for patients who want to be as proactive as possible, warfarin pharmacogenetic testing offers the ability to participate in this decision-making and to potentially reduce their risk of adverse drug events. And in view of the evidence and FDA recommendations, we propose that the discussion with our patients is not whether we should consider pharmacogenetic testing, but that we have considered pharmacogenetic testing, and why we have decided for or against it.

References
  1. Jacobs LG. Warfarin pharmacology, clinical management, and evaluation of hemorrhagic risk for the elderly. Clin Geriatr Med 2006; 22:1732,viiviii.
  2. Rieder MJ, Reiner AP, Gage BF, et al. Effect of VKORC1 haplotypes on transcriptional regulation and warfarin dose. N Engl J Med 2005; 352:22852293.
  3. Higashi MK, Veenstra DL, Kondo LM, et al. Association between CYP2C9 genetic variants and anticoagulation-related outcomes during warfarin therapy. JAMA 2002; 287:16901698.
  4. Shehab N, Sperling LS, Kegler SR, Budnitz DS. National estimates of emergency department visits for hemorrhage-related adverse events from clopidogrel plus aspirin and from warfarin. Arch Intern Med 2010; 170:19261933.
  5. Holbrook A, Schulman S, Witt DM, et al; American College of Chest Physicians. Evidence-based management of anticoagulant therapy: antithrombotic therapy and prevention of thrombosis, 9th ed: American College of Chest Physicians Evidence-Based Clinical Practice Guidelines. Chest 2012; 141(suppl 2):e152Se184S.
  6. Gage BF, Eby C, Johnson JA, et al. Use of pharmacogenetic and clinical factors to predict the therapeutic dose of warfarin. Clin Pharmacol Ther 2008; 84:326331.
  7. Cavallari LH, Shin J, Perera MA. Role of pharmacogenomics in the management of traditional and novel oral anticoagulants. Pharmacotherapy 2011; 31:11921207.
  8. Johnson JA, Gong L, Whirl-Carillo M, et al; Clinical Pharmacogenetics Implementation Consortium. Clinical Pharmacogenetics Implementation Consortium guidelines for CYP2C9 and VKORC1 genotypes and warfarin dosing. Clin Pharmacol Ther 2011; 90:625629.
  9. Limdi NA, McGwin G, Goldstein JA, et al. Influence of CYP2C9 and VKORC1 1173C/T genotype on the risk of hemorrhagic complications in African-American and European-American patients on warfarin. Clin Pharmacol Ther 2008; 83:312321.
  10. Anderson JL, Horne BD, Stevens SM, et al. A randomized and clinical effectiveness trial comparing two pharmacogenetic algorithms and standard care for individualizing warfarin dosing (CoumaGen-II). Circulation 2012; 125:19972005.
  11. Yip VL, Pirmohamed M. Expanding role of pharmacogenomics in the management of cardiovascular disorders. Am J Cardiovasc Drugs 2013; 12 Apr; Epub ahead of print.
  12. Epstein RS, Moyer TP, Aubert RE, et al. Warfarin genotyping reduces hospitalization rates: results from the MM-WES (Medco-Mayo Warfarin Effectiveness Study). J Am Coll Cardiol 2010; 55:28042812.
  13. Wang L, McLeod HL, Weinshilboum RM. Genomics and drug response. N Engl J Med 2011; 364:11441153.
  14. Kitzmiller JP, Groen DK, Phelps MA, Sadee W. Pharmacogenomic testing: relevance in medical practice: why drugs work in some patients but not in others. Cleve Clin J Med 2011; 78:243257.
  15. Carlquist JF, Anderson JL. Using pharmacogenetics in real time to guide warfarin initiation: a clinician update. Circulation 2011; 124:25542559.
Article PDF
Author and Disclosure Information

Mary Rouse, MPH
Center for Personalized Healthcare, Cleveland Clinic

Cari Cristiani, PharmD, BCPS, BCACP
Department of Pharmacy, Cleveland Clinic

Kathryn A. Teng, MD, FACP
Director, Center for Personalized Healthcare, Cleveland Clinic; Assistant Professor of Medicine, Cleveland Clinic Lerner College of Medicine of Case Western Reserve University, Cleveland, OH

Address: Kathryn Teng, MD, FACP, Cleveland Clinic, Center for Personalized Healthcare, 9500 Euclid Avenue, NE5-203, Cleveland, OH 44195; e-mail: tengk@ccf.org

Dr. Teng has disclosed consulting for the Natural Molecular Testing Corporation.

Issue
Cleveland Clinic Journal of Medicine - 80(8)
Publications
Topics
Page Number
483-486
Sections
Author and Disclosure Information

Mary Rouse, MPH
Center for Personalized Healthcare, Cleveland Clinic

Cari Cristiani, PharmD, BCPS, BCACP
Department of Pharmacy, Cleveland Clinic

Kathryn A. Teng, MD, FACP
Director, Center for Personalized Healthcare, Cleveland Clinic; Assistant Professor of Medicine, Cleveland Clinic Lerner College of Medicine of Case Western Reserve University, Cleveland, OH

Address: Kathryn Teng, MD, FACP, Cleveland Clinic, Center for Personalized Healthcare, 9500 Euclid Avenue, NE5-203, Cleveland, OH 44195; e-mail: tengk@ccf.org

Dr. Teng has disclosed consulting for the Natural Molecular Testing Corporation.

Author and Disclosure Information

Mary Rouse, MPH
Center for Personalized Healthcare, Cleveland Clinic

Cari Cristiani, PharmD, BCPS, BCACP
Department of Pharmacy, Cleveland Clinic

Kathryn A. Teng, MD, FACP
Director, Center for Personalized Healthcare, Cleveland Clinic; Assistant Professor of Medicine, Cleveland Clinic Lerner College of Medicine of Case Western Reserve University, Cleveland, OH

Address: Kathryn Teng, MD, FACP, Cleveland Clinic, Center for Personalized Healthcare, 9500 Euclid Avenue, NE5-203, Cleveland, OH 44195; e-mail: tengk@ccf.org

Dr. Teng has disclosed consulting for the Natural Molecular Testing Corporation.

Article PDF
Article PDF
Related Articles

The answer is not clear. There is evidence in favor of pharmacogenetic testing, but not yet enough to strongly recommend it. However, we do believe that physicians should consider it when starting patients on warfarin therapy.

See related commentary

WARFARIN HAS A NARROW THERAPEUTIC WINDOW

Although newer drugs are available, warfarin is still the most commonly used oral anticoagulant for preventing and treating thromboembolism.1 It is highly effective but has a narrow therapeutic window and wide interindividual variability in dosage requirements, which poses challenges to achieving adequate anticoagulation.1–3 Inappropriate dosing contributes to a high rate of bleeding events and emergency room visits.4

Warfarin is monitored using the prothrombin time. Because the prothrombin time varies depending on the assay used, the standardized value called the international normalized ratio (INR) is more commonly used.

Clinical factors such as age, body size, and drug interactions affect warfarin dosage requirements and are important to consider,5 even though they account for only 15% to 20% of the variability in warfarin dose.6

Genetic factors also affect warfarin dosage requirements. The combination of genetic and clinical factors accounts for up to 47% of the dose variability.7

GENES THAT AFFECT WARFARIN

Several genes are known to influence warfarin’s pharmacokinetics and pharmacodynamics. Of these, the two most clinically relevant and well studied are CYP2C9 (which codes for cytochrome P450 2C9) and VKORC1 (which codes for vitamin K epoxide reductase).7 These genes are polymorphic, with some variants producing less-active enzymes that allow warfarin to be more active. Therefore, patients who carry these variants need lower doses of this drug (see below).

CYP2C9 variants

The CYP2C9 gene has several variants. Of these, CYP2C9*2 and CYP2C9*3 are associated with the lowest enzyme activity.

Patients with either of these variants require significantly lower warfarin doses to reach therapeutic levels than those with the wild-type gene (ie, CYP2C9*1). CYP2C9*2 reduces warfarin clearance by 40%, and the CYP2C9*3 variant reduces it by 75%.7 Having a *2 or *3 allele increases the risk of bleeding during warfarin therapy and the time needed to achieve a stable dose.8 Other variants associated with lower warfarin dose requirements are *5, *6, and *11.

The prevalence of these variants is significantly higher in people of European ancestry (roughly one-third) than in Asian people and African Americans,7 although no one has recommended not testing in these low-prevalence populations. Limdi et al9 reported that by including the *5, *6, and *11 variants in genetic testing (in addition to *2 and *3), they could identify more African Americans (9.7%) who carried at least one of these abnormal variants than reported previously. Differences among ethnic groups need to be taken into account when interpreting pharmacogenetic studies.

VKORC1 variants

Patients also need lower doses of warfarin if they carry the VKORC1 −1639G>A variant, and they spend more time with an INR above the therapeutic range and have higher overall INR values. However, having this variant does not appear to increase the risk of bleeding.

The −1639G>A variant is the most common variant of VKORC1. Rarer ones have also been described, but most commercially available tests do not detect them.

Racial differences exist in the prevalence rates of the various VKORC1 polymorphisms, with the most sensitive (low-dose) genotype predominating in Asians and the least sensitive (high-dose) genotype predominating in African Americans. Over 50% of people of European ancestry carry the intermediate-sensitivity genotype (typical dose).7

CURRENT RECOMMENDATIONS FOR OR AGAINST TESTING

FDA labeling

In 2007, the US Food and Drug Administration (FDA) required that the warfarin package insert carry information about initial dosing based on CYP2C9 and VKORC1 testing. This recommendation was revised in 2010 to include a table to help clinicians select an initial warfarin dose if CYP2C9 and VKORC1 genotype information is available. However, the FDA does not require pharmacogenetic testing, leaving the decision to the discretion of the clinician.7

American College of Chest Physicians

The American College of Chest Physicians recommends against routine pharmacogenetic testing (grade 1B) because of a lack of evidence that it improves clinical end points or that it is cost-effective.5

WHAT EVIDENCE SUPORTS GENETIC TESTING TO GUIDE WARFARIN THERAPY?

To date, no large randomized, controlled trial has been published that looked at clinical outcomes with warfarin dosing based on pharmacogenetic testing. However, several smaller studies have suggested it is beneficial.

One trial found that when dosing was informed by pharmacogenetic testing, patients had significantly more time in the therapeutic range, a lower percentage of INRs greater than 4 or less than 1.5, and fewer serious adverse events (death, myocardial infarction, stroke, thromboembolism, and clinically significant bleeding events).10 Patients whose dosage was determined using pharmacogenetic algorithms as opposed to traditional clinical algorithms maintained a therapeutic INR more consistently.11

In addition, compared with historical controls, patients whose physician used pharmacogenetic testing to guide warfarin dosing had a rate of hospitalization 31% lower and a rate of hospitalization specifically for bleeding or thromboembolism 28% lower during 6 months of follow-up.12,13

Several studies have attempted to assess the cost-effectiveness and utility of pharmacogenetic testing in warfarin therapy. As yet, the results have been inconclusive.14 Larger prospective trials are under way and are estimated to be completed in late 2013.15 These include:

  • COAG (Clarification of Optimal Anticoagulation Through Genetics)
  • GIFT (Genetics Informatics Trial of Warfarin to Prevent Venous Thrombosis)
  • EU-PACT (European Pharmacogenetics of Anticoagulant Therapy-Warfarin).

We hope these studies will provide greater clarity on the clinical utility and cost-effectiveness of pharmacogenetic testing to guide warfarin dosing.

 

 

HOW SHOULD GENETIC INFORMATION BE USED TO GUIDE OR ALTER THERAPY?

Algorithms are available for estimating initial and maintenance warfarin doses based on genetic information (CYP2C9 and VKORC1), race or ethnicity, age, sex, body mass index, smoking status, and other medications taken. In addition, models incorporating the INR on day 4 and days 6 to 11 have been developed for dose refinement.15 The algorithms explain 30% to 60% of the variability of the data, with lower values for African Americans.7

A well-developed dosing model that includes traditional clinical factors and patient genetic status is publicly available online at www.warfarindosing.org.4

CPIC: A leader in applied pharmacogenetics

In late 2009, PharmGKB joined forces with the Pharmacogenomics Research Network of the National Institutes of Health to form the Clinical Pharmacogenetics Implementation Consortium (CPIC). This organization issues guidelines that are written by expert clinicians and scientists and then are peer-reviewed, published in leading journals, and simultaneously posted to the PharmGKB website along with supplemental information and updates.

CPIC’s goal is to review the current evidence and to address barriers to the adoption of pharmacogenetic testing into clinical practice. Its guidelines do not advise when or which pharmacogenetic tests should be ordered. Rather, they provide guidance on interpreting and applying such testing, should the test results be available.7

CPIC has guidelines on CYP2C9 and VKORC1 genotypes and warfarin dosing.8 If a patient’s genetic information is available, CPIC strongly recommends the use of pharmacogenetic algorithm-based dosing. If such an algorithm is not accessible, use of a genotype dosing table is recommended.8

Monitoring is still needed

Many factors can affect an individual’s response to warfarin above and beyond the above-noted clinical and genetic traits. These include diet, concomitant medications (both prescription and over-the-counter and herbal), and disease state. There may also be additional genetic polymorphisms not yet identified in various racial and ethnic groups that may affect dosing requirements. And as with all medications, patient compliance and dosing errors have a large potential to affect individual response. Therefore, clinicians should still be diligent about clinical monitoring.15

Most useful for initial dose

As with most pharmacogenetic information, the greatest benefit can be achieved when this information is used to guide the initial dose, although there is also some effect noted when this information is known and acted upon into the 2nd week of treatment.8

Patients on long-term warfarin treatment with stable doses and those unable to achieve stable dosing because of variable adherence or dietary vitamin K intake are less likely to benefit from genetic testing.

There are no published guidelines on the utility of pharmacogenetic testing if a patient is already on a stable dose of warfarin or has a known historical stable dose. There are also no published guidelines on changing the frequency of monitoring based on known genotype.

In children, the data are sparse at this time regarding the utility of pharmacogenetically informed dosing.

HOW DOES ONE ORDER TESTING, AND WHAT IS THE COST?

The FDA has approved four warfarin pharmacogenetic test kits. To be used in clinical decision-making, these tests must be done in a laboratory certified by the Clinical Laboratory Improvement Amendments (CLIA) program.

Testing typically costs a few hundred dollars and may take days for results to be returned if not available on site.15 At Cleveland Clinic, CYP2C9 and VKORC1 testing can be run in-house at a cost of about $700. Generally, many third-party payers do not reimburse for testing without a prior-approval process.

TO TEST OR NOT TO TEST

Pharmacogenetic testing is available and may help optimize warfarin dosing early in treatment, as well as help maintain therapeutic INRs more consistently. There is preliminary evidence that using this information to guide dosing improves clinical outcomes. Several large trials are under way to address additional questions of clinical utility, with results expected in the next year. There are also readily available decision-support tools to guide therapeutic dosing, and when pharmacogenetic test results are available, utilization of a warfarin dosing algorithm is recommended.

The largest barrier remaining appears to be cost (relative to perceived benefit), and until larger trials of clinical utility and cost-effectiveness are completed and analyzed, hurdles exist to obtaining coverage for such testing.

If it is readily available (and can be paid for by insurance companies or out-of-pocket) and test results can be obtained within 24 to 48 hours or before prescribing, pharmacogenetic testing can be a valuable tool to guide and manage warfarin dosing. Particularly for patients who want to be as proactive as possible, warfarin pharmacogenetic testing offers the ability to participate in this decision-making and to potentially reduce their risk of adverse drug events. And in view of the evidence and FDA recommendations, we propose that the discussion with our patients is not whether we should consider pharmacogenetic testing, but that we have considered pharmacogenetic testing, and why we have decided for or against it.

The answer is not clear. There is evidence in favor of pharmacogenetic testing, but not yet enough to strongly recommend it. However, we do believe that physicians should consider it when starting patients on warfarin therapy.

See related commentary

WARFARIN HAS A NARROW THERAPEUTIC WINDOW

Although newer drugs are available, warfarin is still the most commonly used oral anticoagulant for preventing and treating thromboembolism.1 It is highly effective but has a narrow therapeutic window and wide interindividual variability in dosage requirements, which poses challenges to achieving adequate anticoagulation.1–3 Inappropriate dosing contributes to a high rate of bleeding events and emergency room visits.4

Warfarin is monitored using the prothrombin time. Because the prothrombin time varies depending on the assay used, the standardized value called the international normalized ratio (INR) is more commonly used.

Clinical factors such as age, body size, and drug interactions affect warfarin dosage requirements and are important to consider,5 even though they account for only 15% to 20% of the variability in warfarin dose.6

Genetic factors also affect warfarin dosage requirements. The combination of genetic and clinical factors accounts for up to 47% of the dose variability.7

GENES THAT AFFECT WARFARIN

Several genes are known to influence warfarin’s pharmacokinetics and pharmacodynamics. Of these, the two most clinically relevant and well studied are CYP2C9 (which codes for cytochrome P450 2C9) and VKORC1 (which codes for vitamin K epoxide reductase).7 These genes are polymorphic, with some variants producing less-active enzymes that allow warfarin to be more active. Therefore, patients who carry these variants need lower doses of this drug (see below).

CYP2C9 variants

The CYP2C9 gene has several variants. Of these, CYP2C9*2 and CYP2C9*3 are associated with the lowest enzyme activity.

Patients with either of these variants require significantly lower warfarin doses to reach therapeutic levels than those with the wild-type gene (ie, CYP2C9*1). CYP2C9*2 reduces warfarin clearance by 40%, and the CYP2C9*3 variant reduces it by 75%.7 Having a *2 or *3 allele increases the risk of bleeding during warfarin therapy and the time needed to achieve a stable dose.8 Other variants associated with lower warfarin dose requirements are *5, *6, and *11.

The prevalence of these variants is significantly higher in people of European ancestry (roughly one-third) than in Asian people and African Americans,7 although no one has recommended not testing in these low-prevalence populations. Limdi et al9 reported that by including the *5, *6, and *11 variants in genetic testing (in addition to *2 and *3), they could identify more African Americans (9.7%) who carried at least one of these abnormal variants than reported previously. Differences among ethnic groups need to be taken into account when interpreting pharmacogenetic studies.

VKORC1 variants

Patients also need lower doses of warfarin if they carry the VKORC1 −1639G>A variant, and they spend more time with an INR above the therapeutic range and have higher overall INR values. However, having this variant does not appear to increase the risk of bleeding.

The −1639G>A variant is the most common variant of VKORC1. Rarer ones have also been described, but most commercially available tests do not detect them.

Racial differences exist in the prevalence rates of the various VKORC1 polymorphisms, with the most sensitive (low-dose) genotype predominating in Asians and the least sensitive (high-dose) genotype predominating in African Americans. Over 50% of people of European ancestry carry the intermediate-sensitivity genotype (typical dose).7

CURRENT RECOMMENDATIONS FOR OR AGAINST TESTING

FDA labeling

In 2007, the US Food and Drug Administration (FDA) required that the warfarin package insert carry information about initial dosing based on CYP2C9 and VKORC1 testing. This recommendation was revised in 2010 to include a table to help clinicians select an initial warfarin dose if CYP2C9 and VKORC1 genotype information is available. However, the FDA does not require pharmacogenetic testing, leaving the decision to the discretion of the clinician.7

American College of Chest Physicians

The American College of Chest Physicians recommends against routine pharmacogenetic testing (grade 1B) because of a lack of evidence that it improves clinical end points or that it is cost-effective.5

WHAT EVIDENCE SUPORTS GENETIC TESTING TO GUIDE WARFARIN THERAPY?

To date, no large randomized, controlled trial has been published that looked at clinical outcomes with warfarin dosing based on pharmacogenetic testing. However, several smaller studies have suggested it is beneficial.

One trial found that when dosing was informed by pharmacogenetic testing, patients had significantly more time in the therapeutic range, a lower percentage of INRs greater than 4 or less than 1.5, and fewer serious adverse events (death, myocardial infarction, stroke, thromboembolism, and clinically significant bleeding events).10 Patients whose dosage was determined using pharmacogenetic algorithms as opposed to traditional clinical algorithms maintained a therapeutic INR more consistently.11

In addition, compared with historical controls, patients whose physician used pharmacogenetic testing to guide warfarin dosing had a rate of hospitalization 31% lower and a rate of hospitalization specifically for bleeding or thromboembolism 28% lower during 6 months of follow-up.12,13

Several studies have attempted to assess the cost-effectiveness and utility of pharmacogenetic testing in warfarin therapy. As yet, the results have been inconclusive.14 Larger prospective trials are under way and are estimated to be completed in late 2013.15 These include:

  • COAG (Clarification of Optimal Anticoagulation Through Genetics)
  • GIFT (Genetics Informatics Trial of Warfarin to Prevent Venous Thrombosis)
  • EU-PACT (European Pharmacogenetics of Anticoagulant Therapy-Warfarin).

We hope these studies will provide greater clarity on the clinical utility and cost-effectiveness of pharmacogenetic testing to guide warfarin dosing.

 

 

HOW SHOULD GENETIC INFORMATION BE USED TO GUIDE OR ALTER THERAPY?

Algorithms are available for estimating initial and maintenance warfarin doses based on genetic information (CYP2C9 and VKORC1), race or ethnicity, age, sex, body mass index, smoking status, and other medications taken. In addition, models incorporating the INR on day 4 and days 6 to 11 have been developed for dose refinement.15 The algorithms explain 30% to 60% of the variability of the data, with lower values for African Americans.7

A well-developed dosing model that includes traditional clinical factors and patient genetic status is publicly available online at www.warfarindosing.org.4

CPIC: A leader in applied pharmacogenetics

In late 2009, PharmGKB joined forces with the Pharmacogenomics Research Network of the National Institutes of Health to form the Clinical Pharmacogenetics Implementation Consortium (CPIC). This organization issues guidelines that are written by expert clinicians and scientists and then are peer-reviewed, published in leading journals, and simultaneously posted to the PharmGKB website along with supplemental information and updates.

CPIC’s goal is to review the current evidence and to address barriers to the adoption of pharmacogenetic testing into clinical practice. Its guidelines do not advise when or which pharmacogenetic tests should be ordered. Rather, they provide guidance on interpreting and applying such testing, should the test results be available.7

CPIC has guidelines on CYP2C9 and VKORC1 genotypes and warfarin dosing.8 If a patient’s genetic information is available, CPIC strongly recommends the use of pharmacogenetic algorithm-based dosing. If such an algorithm is not accessible, use of a genotype dosing table is recommended.8

Monitoring is still needed

Many factors can affect an individual’s response to warfarin above and beyond the above-noted clinical and genetic traits. These include diet, concomitant medications (both prescription and over-the-counter and herbal), and disease state. There may also be additional genetic polymorphisms not yet identified in various racial and ethnic groups that may affect dosing requirements. And as with all medications, patient compliance and dosing errors have a large potential to affect individual response. Therefore, clinicians should still be diligent about clinical monitoring.15

Most useful for initial dose

As with most pharmacogenetic information, the greatest benefit can be achieved when this information is used to guide the initial dose, although there is also some effect noted when this information is known and acted upon into the 2nd week of treatment.8

Patients on long-term warfarin treatment with stable doses and those unable to achieve stable dosing because of variable adherence or dietary vitamin K intake are less likely to benefit from genetic testing.

There are no published guidelines on the utility of pharmacogenetic testing if a patient is already on a stable dose of warfarin or has a known historical stable dose. There are also no published guidelines on changing the frequency of monitoring based on known genotype.

In children, the data are sparse at this time regarding the utility of pharmacogenetically informed dosing.

HOW DOES ONE ORDER TESTING, AND WHAT IS THE COST?

The FDA has approved four warfarin pharmacogenetic test kits. To be used in clinical decision-making, these tests must be done in a laboratory certified by the Clinical Laboratory Improvement Amendments (CLIA) program.

Testing typically costs a few hundred dollars and may take days for results to be returned if not available on site.15 At Cleveland Clinic, CYP2C9 and VKORC1 testing can be run in-house at a cost of about $700. Generally, many third-party payers do not reimburse for testing without a prior-approval process.

TO TEST OR NOT TO TEST

Pharmacogenetic testing is available and may help optimize warfarin dosing early in treatment, as well as help maintain therapeutic INRs more consistently. There is preliminary evidence that using this information to guide dosing improves clinical outcomes. Several large trials are under way to address additional questions of clinical utility, with results expected in the next year. There are also readily available decision-support tools to guide therapeutic dosing, and when pharmacogenetic test results are available, utilization of a warfarin dosing algorithm is recommended.

The largest barrier remaining appears to be cost (relative to perceived benefit), and until larger trials of clinical utility and cost-effectiveness are completed and analyzed, hurdles exist to obtaining coverage for such testing.

If it is readily available (and can be paid for by insurance companies or out-of-pocket) and test results can be obtained within 24 to 48 hours or before prescribing, pharmacogenetic testing can be a valuable tool to guide and manage warfarin dosing. Particularly for patients who want to be as proactive as possible, warfarin pharmacogenetic testing offers the ability to participate in this decision-making and to potentially reduce their risk of adverse drug events. And in view of the evidence and FDA recommendations, we propose that the discussion with our patients is not whether we should consider pharmacogenetic testing, but that we have considered pharmacogenetic testing, and why we have decided for or against it.

References
  1. Jacobs LG. Warfarin pharmacology, clinical management, and evaluation of hemorrhagic risk for the elderly. Clin Geriatr Med 2006; 22:1732,viiviii.
  2. Rieder MJ, Reiner AP, Gage BF, et al. Effect of VKORC1 haplotypes on transcriptional regulation and warfarin dose. N Engl J Med 2005; 352:22852293.
  3. Higashi MK, Veenstra DL, Kondo LM, et al. Association between CYP2C9 genetic variants and anticoagulation-related outcomes during warfarin therapy. JAMA 2002; 287:16901698.
  4. Shehab N, Sperling LS, Kegler SR, Budnitz DS. National estimates of emergency department visits for hemorrhage-related adverse events from clopidogrel plus aspirin and from warfarin. Arch Intern Med 2010; 170:19261933.
  5. Holbrook A, Schulman S, Witt DM, et al; American College of Chest Physicians. Evidence-based management of anticoagulant therapy: antithrombotic therapy and prevention of thrombosis, 9th ed: American College of Chest Physicians Evidence-Based Clinical Practice Guidelines. Chest 2012; 141(suppl 2):e152Se184S.
  6. Gage BF, Eby C, Johnson JA, et al. Use of pharmacogenetic and clinical factors to predict the therapeutic dose of warfarin. Clin Pharmacol Ther 2008; 84:326331.
  7. Cavallari LH, Shin J, Perera MA. Role of pharmacogenomics in the management of traditional and novel oral anticoagulants. Pharmacotherapy 2011; 31:11921207.
  8. Johnson JA, Gong L, Whirl-Carillo M, et al; Clinical Pharmacogenetics Implementation Consortium. Clinical Pharmacogenetics Implementation Consortium guidelines for CYP2C9 and VKORC1 genotypes and warfarin dosing. Clin Pharmacol Ther 2011; 90:625629.
  9. Limdi NA, McGwin G, Goldstein JA, et al. Influence of CYP2C9 and VKORC1 1173C/T genotype on the risk of hemorrhagic complications in African-American and European-American patients on warfarin. Clin Pharmacol Ther 2008; 83:312321.
  10. Anderson JL, Horne BD, Stevens SM, et al. A randomized and clinical effectiveness trial comparing two pharmacogenetic algorithms and standard care for individualizing warfarin dosing (CoumaGen-II). Circulation 2012; 125:19972005.
  11. Yip VL, Pirmohamed M. Expanding role of pharmacogenomics in the management of cardiovascular disorders. Am J Cardiovasc Drugs 2013; 12 Apr; Epub ahead of print.
  12. Epstein RS, Moyer TP, Aubert RE, et al. Warfarin genotyping reduces hospitalization rates: results from the MM-WES (Medco-Mayo Warfarin Effectiveness Study). J Am Coll Cardiol 2010; 55:28042812.
  13. Wang L, McLeod HL, Weinshilboum RM. Genomics and drug response. N Engl J Med 2011; 364:11441153.
  14. Kitzmiller JP, Groen DK, Phelps MA, Sadee W. Pharmacogenomic testing: relevance in medical practice: why drugs work in some patients but not in others. Cleve Clin J Med 2011; 78:243257.
  15. Carlquist JF, Anderson JL. Using pharmacogenetics in real time to guide warfarin initiation: a clinician update. Circulation 2011; 124:25542559.
References
  1. Jacobs LG. Warfarin pharmacology, clinical management, and evaluation of hemorrhagic risk for the elderly. Clin Geriatr Med 2006; 22:1732,viiviii.
  2. Rieder MJ, Reiner AP, Gage BF, et al. Effect of VKORC1 haplotypes on transcriptional regulation and warfarin dose. N Engl J Med 2005; 352:22852293.
  3. Higashi MK, Veenstra DL, Kondo LM, et al. Association between CYP2C9 genetic variants and anticoagulation-related outcomes during warfarin therapy. JAMA 2002; 287:16901698.
  4. Shehab N, Sperling LS, Kegler SR, Budnitz DS. National estimates of emergency department visits for hemorrhage-related adverse events from clopidogrel plus aspirin and from warfarin. Arch Intern Med 2010; 170:19261933.
  5. Holbrook A, Schulman S, Witt DM, et al; American College of Chest Physicians. Evidence-based management of anticoagulant therapy: antithrombotic therapy and prevention of thrombosis, 9th ed: American College of Chest Physicians Evidence-Based Clinical Practice Guidelines. Chest 2012; 141(suppl 2):e152Se184S.
  6. Gage BF, Eby C, Johnson JA, et al. Use of pharmacogenetic and clinical factors to predict the therapeutic dose of warfarin. Clin Pharmacol Ther 2008; 84:326331.
  7. Cavallari LH, Shin J, Perera MA. Role of pharmacogenomics in the management of traditional and novel oral anticoagulants. Pharmacotherapy 2011; 31:11921207.
  8. Johnson JA, Gong L, Whirl-Carillo M, et al; Clinical Pharmacogenetics Implementation Consortium. Clinical Pharmacogenetics Implementation Consortium guidelines for CYP2C9 and VKORC1 genotypes and warfarin dosing. Clin Pharmacol Ther 2011; 90:625629.
  9. Limdi NA, McGwin G, Goldstein JA, et al. Influence of CYP2C9 and VKORC1 1173C/T genotype on the risk of hemorrhagic complications in African-American and European-American patients on warfarin. Clin Pharmacol Ther 2008; 83:312321.
  10. Anderson JL, Horne BD, Stevens SM, et al. A randomized and clinical effectiveness trial comparing two pharmacogenetic algorithms and standard care for individualizing warfarin dosing (CoumaGen-II). Circulation 2012; 125:19972005.
  11. Yip VL, Pirmohamed M. Expanding role of pharmacogenomics in the management of cardiovascular disorders. Am J Cardiovasc Drugs 2013; 12 Apr; Epub ahead of print.
  12. Epstein RS, Moyer TP, Aubert RE, et al. Warfarin genotyping reduces hospitalization rates: results from the MM-WES (Medco-Mayo Warfarin Effectiveness Study). J Am Coll Cardiol 2010; 55:28042812.
  13. Wang L, McLeod HL, Weinshilboum RM. Genomics and drug response. N Engl J Med 2011; 364:11441153.
  14. Kitzmiller JP, Groen DK, Phelps MA, Sadee W. Pharmacogenomic testing: relevance in medical practice: why drugs work in some patients but not in others. Cleve Clin J Med 2011; 78:243257.
  15. Carlquist JF, Anderson JL. Using pharmacogenetics in real time to guide warfarin initiation: a clinician update. Circulation 2011; 124:25542559.
Issue
Cleveland Clinic Journal of Medicine - 80(8)
Issue
Cleveland Clinic Journal of Medicine - 80(8)
Page Number
483-486
Page Number
483-486
Publications
Publications
Topics
Article Type
Display Headline
Should we use pharmacogenetic testing when prescribing warfarin?
Display Headline
Should we use pharmacogenetic testing when prescribing warfarin?
Sections
Disallow All Ads
Alternative CME
Article PDF Media

Personalizing patient care

Article Type
Changed
Thu, 03/28/2019 - 16:20
Display Headline
Personalizing patient care

The concept and promise of personalized health care have been anticipated for decades. Yet in its breadth and in the way we would like it practiced, it is in its infancy.

Personalized health care aims to individualize care by integrating a person’s unique clinical, molecular (ie, genetic, genomic), and environmental information. Applied not only when the patient is sick but also when he or she is well, it builds on and enhances our current standards of care.

As Sir William Osler recognized more than a century ago, “Variability is the law of life, and as no two faces are the same, so no two bodies are alike, and no two individuals react alike and behave alike under the abnormal conditions which we know as disease.”1

PREDICTING THE RISK OF DISEASE

For years, we have attempted to predict and stratify the risk of disease, and the Human Genome Project has given us a new set of tools to help understand the complexity of disease and its variability.

We know and have known for decades—and in some cultures, for centuries—that family history is the most clinically validated tool for predicting the risk of disease.

Nevertheless, evidence suggests that we physicians are not collecting adequate family histories, and because of this, we are missing opportunities to intervene and prevent diseases predicted by the family history. The current standard of care is to use family medical histories to hone genetic differential diagnoses, and based on the differential diagnoses, to target specific genes to test in the setting of genetic counseling. Current genetic testing is used for molecular diagnoses and predictive testing so that gene-specific clinical management can be subsequently tailored.

PREDICTING RESPONSE TO TREATMENT

The use of personalized health care to predict response to treatment is a novel and constantly evolving practice.

ABO blood typing is a form of genetics-based personalization of safe transfusion that dates back to World War II.

A prominent, recent success story is in cancer treatment. For example, the American Society of Clinical Oncology now recommends that tumors from patients with node-negative, estrogen-receptor-positive breast cancer be evaluated with the Oncotype DX assay.2 This test measures the expression of 21 genes, and the score obtained identifies patients most likely to benefit from adjuvant chemotherapy. A similar 12-gene expression signature has been developed for colon cancer, and others have been developed for hematologic cancers.2,3 As with other new but apparently valid tests, the risk scores derived are sensitive at the extremes but ambiguous in the mid-ranges. We anticipate many more developments in this field.

In the field of pharmacogenomics, there is evidence to suggest that prior knowledge of CYP2C9 and VKORC1 genotypes enhances outcomes for patients starting treatment with warfarin. The US Food and Drug Administration revised the label on warfarin in February 2010, suggesting that genotypes be taken into consideration when the drug is prescribed.4

However, clinicians have been slow to adopt genotype testing when prescribing warfarin. Some cite the paucity of large, randomized, controlled trials demonstrating clinical utility of genotype-informed prescribing. Others cite concern that warfarin will soon become obsolete with the arrival of newer anticoagulants (such as factor X inhibitors) that do not carry warfarin’s adverse effects, and these genotypes will therefore become moot. Perhaps, as we move forward and new drugs are developed, companion genotype tests could be developed at the same time to be used with them.5

IMPROVING CARE, SAVING MONEY, AND EMPOWERING PATIENTS

The goal of personalized health care, by customizing treatments (medication types and dosages) and preventive strategies, is to optimize medical care and improve outcomes for each patient. It could improve the quality of care by targeting interventions and reducing adverse events, topics that are important to all of us in the current environment of health care reform.

A personalized approach might also, in the long run, decrease the cost of health care by driving appropriate utilization of resources.

Lastly, the true value of personalized health care may be in its potential to improve patient satisfaction and to empower our patients to work with us towards better health.

WE LAUNCH A NEW SERIES

To keep physicians up-to-date on progress in personalized health care, the Cleveland Clinic Journal of Medicine will present a series of articles on the topic. The series, to run once a quarter, begins in this issue, on page 331, with an article on the importance of the family history as a piece of genetic information that can help to predict the risk of disease and inform preventive care plans. Future topics will include the role of genetics and genomics in personalized care of patients with breast and colorectal cancers; the genetic counselor as a part of the health care team; pharmacogenomics; and ethical, legal, and societal considerations.

Our goal in this series is to provide practical information to help our readers incorporate personalized approaches into daily practice. In addition, as patients become more interested in and informed about personalized health care, we hope this information will help clinicians to effectively coach them about its potential benefits and risks. We also hope this information will enable our readers to ask the right questions so that patient and health care provider can work together to help the patient grow old gracefully.

As the series unfolds, we ask you to send us feedback and to suggest other topics in personalized health care you would like us to cover in this series.

References
  1. Osler W. Aequanimitas, With Other Addresses to Medical Students, Nurses and Practitioners of Medicine. 2nd edition. Philadelphia, PA: P. Blakiston’s Sone & Co, 1906:348.
  2. McDermott U, Downing JR, Stratton MR. Genomics and the continuum of cancer care. N Engl J Med 2011; 364:340350.
  3. Eng C. Microenvironmental protection in diffuse large-B-cell lymphoma. N Engl J Med 2008; 359:23792381.
  4. Wang L, McLeod HL, Weinshilboum RM. Genomics and drug response. N Engl J Med 2011; 364:11441153.
  5. Hamburg MA, Collins FS. The path to personalized medicine. N Engl J Med 2010; 363:301304.
Article PDF
Author and Disclosure Information

Kathryn A. Teng, MD, FACP
Director, Center for Personalized Healthcare, Cleveland Clinic

Charis Eng, MD, PhD
Hardis Professor, Chair, and Director, Genomic Medicine Institute; Director, Center for Personalized Genetic Healthcare, Cleveland Clinic

Issue
Cleveland Clinic Journal of Medicine - 79(5)
Publications
Topics
Page Number
329-330
Sections
Author and Disclosure Information

Kathryn A. Teng, MD, FACP
Director, Center for Personalized Healthcare, Cleveland Clinic

Charis Eng, MD, PhD
Hardis Professor, Chair, and Director, Genomic Medicine Institute; Director, Center for Personalized Genetic Healthcare, Cleveland Clinic

Author and Disclosure Information

Kathryn A. Teng, MD, FACP
Director, Center for Personalized Healthcare, Cleveland Clinic

Charis Eng, MD, PhD
Hardis Professor, Chair, and Director, Genomic Medicine Institute; Director, Center for Personalized Genetic Healthcare, Cleveland Clinic

Article PDF
Article PDF

The concept and promise of personalized health care have been anticipated for decades. Yet in its breadth and in the way we would like it practiced, it is in its infancy.

Personalized health care aims to individualize care by integrating a person’s unique clinical, molecular (ie, genetic, genomic), and environmental information. Applied not only when the patient is sick but also when he or she is well, it builds on and enhances our current standards of care.

As Sir William Osler recognized more than a century ago, “Variability is the law of life, and as no two faces are the same, so no two bodies are alike, and no two individuals react alike and behave alike under the abnormal conditions which we know as disease.”1

PREDICTING THE RISK OF DISEASE

For years, we have attempted to predict and stratify the risk of disease, and the Human Genome Project has given us a new set of tools to help understand the complexity of disease and its variability.

We know and have known for decades—and in some cultures, for centuries—that family history is the most clinically validated tool for predicting the risk of disease.

Nevertheless, evidence suggests that we physicians are not collecting adequate family histories, and because of this, we are missing opportunities to intervene and prevent diseases predicted by the family history. The current standard of care is to use family medical histories to hone genetic differential diagnoses, and based on the differential diagnoses, to target specific genes to test in the setting of genetic counseling. Current genetic testing is used for molecular diagnoses and predictive testing so that gene-specific clinical management can be subsequently tailored.

PREDICTING RESPONSE TO TREATMENT

The use of personalized health care to predict response to treatment is a novel and constantly evolving practice.

ABO blood typing is a form of genetics-based personalization of safe transfusion that dates back to World War II.

A prominent, recent success story is in cancer treatment. For example, the American Society of Clinical Oncology now recommends that tumors from patients with node-negative, estrogen-receptor-positive breast cancer be evaluated with the Oncotype DX assay.2 This test measures the expression of 21 genes, and the score obtained identifies patients most likely to benefit from adjuvant chemotherapy. A similar 12-gene expression signature has been developed for colon cancer, and others have been developed for hematologic cancers.2,3 As with other new but apparently valid tests, the risk scores derived are sensitive at the extremes but ambiguous in the mid-ranges. We anticipate many more developments in this field.

In the field of pharmacogenomics, there is evidence to suggest that prior knowledge of CYP2C9 and VKORC1 genotypes enhances outcomes for patients starting treatment with warfarin. The US Food and Drug Administration revised the label on warfarin in February 2010, suggesting that genotypes be taken into consideration when the drug is prescribed.4

However, clinicians have been slow to adopt genotype testing when prescribing warfarin. Some cite the paucity of large, randomized, controlled trials demonstrating clinical utility of genotype-informed prescribing. Others cite concern that warfarin will soon become obsolete with the arrival of newer anticoagulants (such as factor X inhibitors) that do not carry warfarin’s adverse effects, and these genotypes will therefore become moot. Perhaps, as we move forward and new drugs are developed, companion genotype tests could be developed at the same time to be used with them.5

IMPROVING CARE, SAVING MONEY, AND EMPOWERING PATIENTS

The goal of personalized health care, by customizing treatments (medication types and dosages) and preventive strategies, is to optimize medical care and improve outcomes for each patient. It could improve the quality of care by targeting interventions and reducing adverse events, topics that are important to all of us in the current environment of health care reform.

A personalized approach might also, in the long run, decrease the cost of health care by driving appropriate utilization of resources.

Lastly, the true value of personalized health care may be in its potential to improve patient satisfaction and to empower our patients to work with us towards better health.

WE LAUNCH A NEW SERIES

To keep physicians up-to-date on progress in personalized health care, the Cleveland Clinic Journal of Medicine will present a series of articles on the topic. The series, to run once a quarter, begins in this issue, on page 331, with an article on the importance of the family history as a piece of genetic information that can help to predict the risk of disease and inform preventive care plans. Future topics will include the role of genetics and genomics in personalized care of patients with breast and colorectal cancers; the genetic counselor as a part of the health care team; pharmacogenomics; and ethical, legal, and societal considerations.

Our goal in this series is to provide practical information to help our readers incorporate personalized approaches into daily practice. In addition, as patients become more interested in and informed about personalized health care, we hope this information will help clinicians to effectively coach them about its potential benefits and risks. We also hope this information will enable our readers to ask the right questions so that patient and health care provider can work together to help the patient grow old gracefully.

As the series unfolds, we ask you to send us feedback and to suggest other topics in personalized health care you would like us to cover in this series.

The concept and promise of personalized health care have been anticipated for decades. Yet in its breadth and in the way we would like it practiced, it is in its infancy.

Personalized health care aims to individualize care by integrating a person’s unique clinical, molecular (ie, genetic, genomic), and environmental information. Applied not only when the patient is sick but also when he or she is well, it builds on and enhances our current standards of care.

As Sir William Osler recognized more than a century ago, “Variability is the law of life, and as no two faces are the same, so no two bodies are alike, and no two individuals react alike and behave alike under the abnormal conditions which we know as disease.”1

PREDICTING THE RISK OF DISEASE

For years, we have attempted to predict and stratify the risk of disease, and the Human Genome Project has given us a new set of tools to help understand the complexity of disease and its variability.

We know and have known for decades—and in some cultures, for centuries—that family history is the most clinically validated tool for predicting the risk of disease.

Nevertheless, evidence suggests that we physicians are not collecting adequate family histories, and because of this, we are missing opportunities to intervene and prevent diseases predicted by the family history. The current standard of care is to use family medical histories to hone genetic differential diagnoses, and based on the differential diagnoses, to target specific genes to test in the setting of genetic counseling. Current genetic testing is used for molecular diagnoses and predictive testing so that gene-specific clinical management can be subsequently tailored.

PREDICTING RESPONSE TO TREATMENT

The use of personalized health care to predict response to treatment is a novel and constantly evolving practice.

ABO blood typing is a form of genetics-based personalization of safe transfusion that dates back to World War II.

A prominent, recent success story is in cancer treatment. For example, the American Society of Clinical Oncology now recommends that tumors from patients with node-negative, estrogen-receptor-positive breast cancer be evaluated with the Oncotype DX assay.2 This test measures the expression of 21 genes, and the score obtained identifies patients most likely to benefit from adjuvant chemotherapy. A similar 12-gene expression signature has been developed for colon cancer, and others have been developed for hematologic cancers.2,3 As with other new but apparently valid tests, the risk scores derived are sensitive at the extremes but ambiguous in the mid-ranges. We anticipate many more developments in this field.

In the field of pharmacogenomics, there is evidence to suggest that prior knowledge of CYP2C9 and VKORC1 genotypes enhances outcomes for patients starting treatment with warfarin. The US Food and Drug Administration revised the label on warfarin in February 2010, suggesting that genotypes be taken into consideration when the drug is prescribed.4

However, clinicians have been slow to adopt genotype testing when prescribing warfarin. Some cite the paucity of large, randomized, controlled trials demonstrating clinical utility of genotype-informed prescribing. Others cite concern that warfarin will soon become obsolete with the arrival of newer anticoagulants (such as factor X inhibitors) that do not carry warfarin’s adverse effects, and these genotypes will therefore become moot. Perhaps, as we move forward and new drugs are developed, companion genotype tests could be developed at the same time to be used with them.5

IMPROVING CARE, SAVING MONEY, AND EMPOWERING PATIENTS

The goal of personalized health care, by customizing treatments (medication types and dosages) and preventive strategies, is to optimize medical care and improve outcomes for each patient. It could improve the quality of care by targeting interventions and reducing adverse events, topics that are important to all of us in the current environment of health care reform.

A personalized approach might also, in the long run, decrease the cost of health care by driving appropriate utilization of resources.

Lastly, the true value of personalized health care may be in its potential to improve patient satisfaction and to empower our patients to work with us towards better health.

WE LAUNCH A NEW SERIES

To keep physicians up-to-date on progress in personalized health care, the Cleveland Clinic Journal of Medicine will present a series of articles on the topic. The series, to run once a quarter, begins in this issue, on page 331, with an article on the importance of the family history as a piece of genetic information that can help to predict the risk of disease and inform preventive care plans. Future topics will include the role of genetics and genomics in personalized care of patients with breast and colorectal cancers; the genetic counselor as a part of the health care team; pharmacogenomics; and ethical, legal, and societal considerations.

Our goal in this series is to provide practical information to help our readers incorporate personalized approaches into daily practice. In addition, as patients become more interested in and informed about personalized health care, we hope this information will help clinicians to effectively coach them about its potential benefits and risks. We also hope this information will enable our readers to ask the right questions so that patient and health care provider can work together to help the patient grow old gracefully.

As the series unfolds, we ask you to send us feedback and to suggest other topics in personalized health care you would like us to cover in this series.

References
  1. Osler W. Aequanimitas, With Other Addresses to Medical Students, Nurses and Practitioners of Medicine. 2nd edition. Philadelphia, PA: P. Blakiston’s Sone & Co, 1906:348.
  2. McDermott U, Downing JR, Stratton MR. Genomics and the continuum of cancer care. N Engl J Med 2011; 364:340350.
  3. Eng C. Microenvironmental protection in diffuse large-B-cell lymphoma. N Engl J Med 2008; 359:23792381.
  4. Wang L, McLeod HL, Weinshilboum RM. Genomics and drug response. N Engl J Med 2011; 364:11441153.
  5. Hamburg MA, Collins FS. The path to personalized medicine. N Engl J Med 2010; 363:301304.
References
  1. Osler W. Aequanimitas, With Other Addresses to Medical Students, Nurses and Practitioners of Medicine. 2nd edition. Philadelphia, PA: P. Blakiston’s Sone & Co, 1906:348.
  2. McDermott U, Downing JR, Stratton MR. Genomics and the continuum of cancer care. N Engl J Med 2011; 364:340350.
  3. Eng C. Microenvironmental protection in diffuse large-B-cell lymphoma. N Engl J Med 2008; 359:23792381.
  4. Wang L, McLeod HL, Weinshilboum RM. Genomics and drug response. N Engl J Med 2011; 364:11441153.
  5. Hamburg MA, Collins FS. The path to personalized medicine. N Engl J Med 2010; 363:301304.
Issue
Cleveland Clinic Journal of Medicine - 79(5)
Issue
Cleveland Clinic Journal of Medicine - 79(5)
Page Number
329-330
Page Number
329-330
Publications
Publications
Topics
Article Type
Display Headline
Personalizing patient care
Display Headline
Personalizing patient care
Sections
Disallow All Ads
Alternative CME
Article PDF Media

A young woman with fatigue

Article Type
Changed
Tue, 04/17/2018 - 14:59
Display Headline
A young woman with fatigue

A 22-year-old woman presents to the clinic for evaluation of fatigue. She has not felt well for the past few years. Her current symptoms include generalized fatigue and diarrhea, characterized as two to three semi-formed, nonbloody bowel movements each day and occasional episodes of watery diarrhea. Her bowel movements are usually precipitated by meals. She consumes a regular diet and has not recognized any intolerance to any particular foods. She denies having any abdominal pain, nausea, vomiting, recent travel, joint pain, rash, or change in the texture of her hair. She has been seen by several internists in her hometown, who have not provided her with a specific diagnosis.

Her medical history is significant for anemia, anxiety, and depression. Menarche occurred at age 16. Her menstrual cycle has been regular, with bleeding noted to be only modest. Her medications include oral contraceptive pills. She has not had previous surgeries.

On examination, she appears well. She is afebrile, weighs 128 lbs, and is 63 inches tall. The physical examination is normal, including a rectal examination and fecal occult blood testing.

Routine laboratory tests are performed. Results:

  • White blood cell count 3.88 × 109/L (normal 4.0–11)
  • Hemoglobin 10.4 g/dL (normal 12–16)
  • Hematocrit 34% (normal 37%–47%)
  • Mean corpuscular volume 80.2 fL (normal 80–100)
  • Mean corpuscular hemoglobin 24.5 pG (normal 27–34)
  • Platelet count 365 × 109/L (normal 150–400)
  • Sodium 141 mmol/L (normal 132–148)
  • Potassium 4.2 mmol/L (normal 3.5–5.0)
  • Chloride 107 mmol/L (normal 98–110)
  • Alanine aminotransferase 22 U/L (normal 0–45)
  • Glucose 66 mg/dL (normal 65–100)
  • Blood urea nitrogen 6 mg/dL (normal 8–25)
  • Creatinine 0.6 mg/dL (normal 0.7–1.4)
  • Thyroid-stimulating hormone 2.860 mIU/L (normal 0.4–5.5)
  • Red blood cell folate 539 ng/mL (normal 257–800)
  • Vitamin B12 321 pg/mL (normal 221–700)
  • Iron/total iron-binding capacity 21/445 μg/dL (normal 30–140, 210–415)
  • Ferritin 5 ng/mL (normal 9–150).

DIFFERENTIAL DIAGNOSIS

1. Which of the following is the most likely cause of her diarrhea?

  • Thyroid disease
  • Functional bowel disease
  • Gluten-sensitive enteropathy (celiac disease)

Given her constellation of symptoms (fatigue, neuropsychiatric changes, iron deficiency anemia, and diarrhea), celiac disease is the most likely diagnosis. Hyperthyroidism can cause diarrhea, but this is unlikely since her thyroid tests are normal. Functional bowel disease is a diagnosis of exclusion and usually has a more chronic, fluctuating course.

CELIAC DISEASE HAS VARIOUS PRESENTATIONS

Celiac disease has various presentations and therefore has been classified into several types1,2:

Classic disease is dominated by symptoms of malabsorption. The diagnosis is established by serologic testing, findings of villous atrophy on biopsy, and improvement of symptoms on a gluten-free diet. However, the presentation of celiac disease has changed, and now atypical presentations are more common in adults (see below). The reason for the change in presentation is not known, but some have hypothesized that it is related to an increase in breast-feeding and the later introduction of cereals into infants’ diets.

Celiac disease with atypical symptoms is characterized by extraintestinal manifestations with few or no gastrointestinal (GI) symptoms. Patients may present with iron-deficiency anemia; osteoporosis or vitamin D deficiency; arthritis; neurologic symptoms such as ataxia, headaches, or depression or anxiety; myocarditis; infertility; or elevated aminotransferase levels. As in classic celiac disease, the diagnosis is established with serologic testing, findings of villous atrophy on biopsy, and improvement of symptoms on a gluten-free diet.

Latent disease includes cases in patients with positive serologic tests but no villous atrophy on biopsy. These patients have no symptoms but may develop symptoms or histologic changes later.

Silent disease refers to cases in patients who have no symptoms but have a positive serologic test and villous atrophy on biopsy. These cases are usually detected via screening of people at high risk, ie, relatives of patients with celiac disease.

It is important that clinicians be aware of the various symptoms and presentations of celiac disease in order to make the diagnosis.

 

 

CONFIRMING CELIAC DISEASE

2. Which of the following is used to test for celiac disease?

  • Immunoglobulin G (IgG) and immunoglobulin A (IgA) antigliadin antibody testing
  • IgA antiendomysial antibody and IgA antitransglutaminase antibody testing
  • HLA DQ2/DQ8 testing

The sensitivity of antigliadin antibody testing is only about 70% to 85%, and its specificity is about 70% to 90%. Better serologic tests are those for IgA antiendomysial and antitransglutaminase antibodies, which have sensitivities greater than 90% and specificities greater than 95%.3 HLA DQ2/DQ8 testing has a high sensitivity (> 90%–95%), but because about 30% of the general population also carry these markers, the specificity of this test is not ideal. This test is best used for its negative predictive value—ie, to rule out the diagnosis of celiac disease.

Of note: 1% to 2% of patients with celiac disease have a deficiency of IgA.4 Therefore, if the clinical suspicion for celiac disease is high but the IgA antibody tests are negative or equivocal, IgG antitransglutaminase and IgG antiendomysial antibody tests can help establish the diagnosis. HLA testing in this situation can also help rule out the diagnosis.

CONFIRMING CELIAC DISEASE—CONTINUED

3. What test should be performed next in this patient?

  • Upper GI series with small-bowel follow-through
  • Esophagogastroduodenoscopy with biopsies
  • Small-bowel barium study
  • Video capsule endoscopy

Today, the presumptive diagnosis of celiac disease requires positive serologic testing and biopsy results. Esophagogastroduodenoscopy with biopsies should be ordered. Upper GI series and barium studies do not provide a tissue diagnosis. Barium studies and other radiologic tests can be considered if a patient does not have the expected response to a strict gluten-free diet or if one suspects complications of celiac disease, such as GI lymphoma.

Video capsule endoscopy is an emerging tool for diagnosing celiac disease, as suggested in several trials.5 Some findings seen on video capsule endoscopy in patients with celiac disease include mosaicism, nodularity, visible vessels, and loss of mucosal folds. However, the role of this test continues to be investigated, and biopsy is still required to confirm the diagnosis.

Figure 1. A low-power view (top left) and high-power view (top right) of the normal villous architecture of the small intestine. The high-power view shows the enterocytes and interspersed goblet cells (arrows). Bottom left, a sample from a patient with celiac disease shows severe villous blunting in this low-power view. Bottom right, a higher-power view shows loss of columnar epithelial shape, mucin depletion, and an increased nuclear-to-cytoplasmic ratio within the epithelial layer. Within the normal epithelial layer, there is an increased ratio of lymphocytes to epithelial cells. The normal ratio of lymphocytes to epithelial cells is 1 to 5–10. In this specimen the ratio is approximately 3 to 5 lymphocytes to 5 epithelial cells. The lamina propria also shows an increased number of plasma cells. This constellation of microscopic findings is characteristic of gluten-sensitive enteropathy.
A definitive diagnosis is confirmed when symptoms resolve with a gluten-free diet, and repeat biopsies (3–9 months after dietary changes) show histologic improvement, although the need for repeat biopsy is controversial. A hallmark histologic finding is villous atrophy (Figure 1). However, villous atrophy may be patchy, and it is recommended that multiple biopsy specimens be taken from the duodenal mucosa to increase the diagnostic yield.

WHO SHOULD BE TESTED FOR CELIAC DISEASE?

The reported prevalence of symptomatic celiac disease is about 1 in 1,000 live births in populations of northern European ancestry, ranging from 1 in 250 (in Sweden) to 1 in 4,000 (in Denmark).6 The prevalence appears to be higher in women than in men.7

In a large US study, the prevalence of celiac disease was 1 in 22 in first-degree relatives of celiac patients, 1 in 39 in second-degree relatives, 1 in 56 in patients with either GI symptoms or a condition associated with celiac disease, and 1 in 133 in groups not at risk.8 Another study found that the prevalence of antiendomysial antibodies in US blood donors was as high as 1 in 2,502.

Given that patients with celiac disease may not present with classic symptoms, it has been suggested that the following groups of patients be tested for it1:

  • Patients with GI symptoms such as chronic diarrhea, malabsorption, weight loss, or abdominal symptoms
  • Patients without diarrhea but with other unexplained signs or symptoms that could be due to celiac disease, such as iron-deficiency anemia, elevated aminotransferase levels, short stature, delayed puberty, or infertility
  • Symptomatic patients at high risk for celiac disease. Risk factors include type 1 diabetes or other autoimmune endocrinopathies, first- and second-degree relatives of people with celiac disease, and patients with Turner, Down, or Williams syndromes.

Screening of the general population is not recommended, even in populations at high risk (eg, white people of northern European ancestry).

 

 

WHAT CAN CELIAC PATIENTS EAT?

4. Patients with celiac disease should avoid eating which of the following?

  • Wheat
  • Barley
  • Rye
  • Oats

Patients with celiac disease should follow a gluten-free diet and should initially eliminate all of these substances.

Some recent studies have suggested that pure oat powder can be tolerated without disease recurrence, although the long-term safety of oat consumption in patients with celiac disease is uncertain.9 It may be reasonable for patients to reintroduce oats when the disease is under control, especially since uncontaminated oats can be obtained from reliable retail or wholesale stores. The definitive diagnosis of celiac disease requires clinical suspicion, serologic tests, biopsy, and documented clinical and histologic improvement after a gluten-free diet is started.

All patients with celiac disease should receive dietary counseling and referral to a nutritionist who is experienced in the treatment of this disease. Because of the significant lifestyle and dietary changes involved in treating this disease, many patients may also benefit from participating in a celiac support group.

COMPLICATIONS OF CELIAC DISEASE

5. What are the complications of untreated celiac disease?

  • Anemia
  • Osteoporosis
  • Intestinal lymphoma
  • Infertility
  • Neuropsychiatric symptoms
  • Rash

All of the above are complications of untreated celiac disease and are often clinical features at presentation. Patients with celiac disease should be tested for anemia and nutritional deficiencies, including iron, folate, calcium, and vitamin D deficiency.

All patients should also undergo dual-energy x-ray absorptiometric scanning. Bone loss is thought to be related to vitamin D deficiency and secondary hyperparathyroidism, and may be partially reversed with a gluten-free diet.

Celiac disease is associated with hyposplenism, so pneumococcal vaccination should be considered. Celiac disease is also frequently associated with the rash of dermatitis herpetiformis, and diagnosis of this rash should prompt an evaluation for celiac disease.

Other associated conditions include Down syndrome, selective IgA deficiency, and other autoimmune diseases such as type 1 diabetes, thyroid disease, and liver disease.

WHAT HAPPENED TO OUR PATIENT?

Our patient tested positive for antiendomysial and antitransglutaminase antibodies and underwent small-bowel biopsy, which confirmed the diagnosis of celiac disease. She was started on a gluten-free diet, and within 2 weeks she noted an improvement in her symptoms of fatigue, GI upset, mood disorders, and difficulty with concentration. She met with a nutritionist who specializes in celiac disease and joined a celiac support group.

However, about 2 months later, her symptoms recurred. She again met with her nutritionist, who confirmed that she was adhering to a gluten-free and lactose-free diet. Even so, when she was tested again for antitransglutaminase antibodies, the titer was elevated. Stool cultures were obtained and were negative. She was started on a course of prednisone, and her symptoms resolved.

WHAT IF PATIENTS DO NOT RESPOND TO TREATMENT?

The most common cause of recurrent symptoms or nonresponse to treatment is noncompliance with the gluten-free diet or inadvertent ingestion of gluten. Patients who do not respond to treatment or who have a period of response but then relapse should be referred back to a nutritionist who specializes in celiac disease.

If a patient continues to have symptoms despite strict adherence to a gluten-free diet, other disorders should be considered, such as concomitant lactose intolerance, small-bowel bacterial overgrowth, pancreatic insufficiency, or irritable bowel syndrome. If these conditions are ruled out, patients can be considered for treatment with prednisone or other immunosuppressive agents. Patients with refractory symptoms are at higher risk of more severe complications of celiac disease, such as intestinal lymphoma, intestinal strictures, and collagenous colitis.

TAKE-HOME POINTS

  • Celiac disease classically presents with symptoms of malabsorption, but nonclassic presentations are much more common.
  • Celiac disease should be tested for in patients with or without symptoms of mal-absorption and other associated signs or symptoms including unexplained iron-deficiency anemia, infertility, short stature, delayed puberty, or elevated transaminases. Testing should be considered for symptomatic patients with type 1 diabetes or other autoimmune endocrinopathies, first- and second-degree relatives of patients with known disease, and those with certain chromosomal abnormalities.
  • Heightened physician awareness is important in the diagnosis of celiac disease.
  • Diagnosis depends on serologic testing, biopsy, and clinical improvement on a gluten-free diet.
  • Treatment should consist of education about the disease, consultation with a nutritionist experienced in celiac disease, and lifelong adherence to a gluten-free diet. Referral to a celiac support group should be considered.
  • Long-term follow-up should include heightened vigilance and awareness of the complications of celiac disease such as osteoporosis, vitamin D deficiency and other nutritional deficiencies, increased risk of malignancy, association with low birth-weight infants and preterm labor, and occurrence of autoimmune disorders.

Acknowledgments: I would like to extend a special thank you to Dr. Walter Henricks, Director, Center for Pathology Informatics, Pathology and Laboratory Medicine, Cleveland Clinic, for providing biopsy slides and interpretation. I would also like to extend thanks to Dr. Derek Abbott, Department of Pathology, Case Western University Hospitals, for his helpful criticisms.

References
  1. National Institutes of Health. NIH Consensus Development Conference on Celiac Disease, 2004 Accessed 1/29/2008. http://consensus.nih.gov/2004/2004CeliacDisease118html.htm.
  2. Rostom A, Murray JA, Kagnoff MF. American Gastroenterological Association (AGA) Institute technical review on the diagnosis and management of celiac disease. Gastroenterology 2006; 131:19812002.
  3. Hellekson K. AHRQ releases practice guidelines for celiac disease screening. Am Fam Phys 2005; 71:13.
  4. Cataldo F, Marino V, Bottaro G, Greco P, Ventura A. Celiac disease and selective immunoglobulin A deficiency. J Pediatr 1997; 131:306308.
  5. Kesari A, Bobba RK, Arsura EL. Video capsule endoscopy and celiac disease. Gastrointest Endosc 2005; 62:796797.
  6. Branski D, Fasano A, Troncone R. Latest developments in the pathogenesis and treatment of celiac disease. J Pediatr 2006; 149:295300.
  7. Rampertab SD, Pooran N, Brar P, Singh P, Green PH. Trends in the presentation of celiac disease. Am J Med 2006; 119 4:355.e9e14.
  8. Fasano A, Berti I, Gerarduzzi T, et al. Prevalence of celiac disease in at-risk and not-at-risk groups in the United States: a large multicenter study. Arch Intern Med 2003; 163:286292.
  9. Janatuinen EK, Pikkarainen PH, Kemppainen TA, et al. A comparison of diets with and without oats in adults with celiac disease. N Engl J Med 1995; 333:10331037.
Article PDF
Author and Disclosure Information

Kathryn A. Teng, MD
Department of General Internal Medicine, Cleveland Clinic

Address: Kathryn A. Teng, MD, Department of General Internal Medicine, S70, Cleveland Clinic, 9500 Euclid Avenue, Cleveland, OH 44195; e-mail tengk@ccf.org

Issue
Cleveland Clinic Journal of Medicine - 75(3)
Publications
Topics
Page Number
215-219
Sections
Author and Disclosure Information

Kathryn A. Teng, MD
Department of General Internal Medicine, Cleveland Clinic

Address: Kathryn A. Teng, MD, Department of General Internal Medicine, S70, Cleveland Clinic, 9500 Euclid Avenue, Cleveland, OH 44195; e-mail tengk@ccf.org

Author and Disclosure Information

Kathryn A. Teng, MD
Department of General Internal Medicine, Cleveland Clinic

Address: Kathryn A. Teng, MD, Department of General Internal Medicine, S70, Cleveland Clinic, 9500 Euclid Avenue, Cleveland, OH 44195; e-mail tengk@ccf.org

Article PDF
Article PDF

A 22-year-old woman presents to the clinic for evaluation of fatigue. She has not felt well for the past few years. Her current symptoms include generalized fatigue and diarrhea, characterized as two to three semi-formed, nonbloody bowel movements each day and occasional episodes of watery diarrhea. Her bowel movements are usually precipitated by meals. She consumes a regular diet and has not recognized any intolerance to any particular foods. She denies having any abdominal pain, nausea, vomiting, recent travel, joint pain, rash, or change in the texture of her hair. She has been seen by several internists in her hometown, who have not provided her with a specific diagnosis.

Her medical history is significant for anemia, anxiety, and depression. Menarche occurred at age 16. Her menstrual cycle has been regular, with bleeding noted to be only modest. Her medications include oral contraceptive pills. She has not had previous surgeries.

On examination, she appears well. She is afebrile, weighs 128 lbs, and is 63 inches tall. The physical examination is normal, including a rectal examination and fecal occult blood testing.

Routine laboratory tests are performed. Results:

  • White blood cell count 3.88 × 109/L (normal 4.0–11)
  • Hemoglobin 10.4 g/dL (normal 12–16)
  • Hematocrit 34% (normal 37%–47%)
  • Mean corpuscular volume 80.2 fL (normal 80–100)
  • Mean corpuscular hemoglobin 24.5 pG (normal 27–34)
  • Platelet count 365 × 109/L (normal 150–400)
  • Sodium 141 mmol/L (normal 132–148)
  • Potassium 4.2 mmol/L (normal 3.5–5.0)
  • Chloride 107 mmol/L (normal 98–110)
  • Alanine aminotransferase 22 U/L (normal 0–45)
  • Glucose 66 mg/dL (normal 65–100)
  • Blood urea nitrogen 6 mg/dL (normal 8–25)
  • Creatinine 0.6 mg/dL (normal 0.7–1.4)
  • Thyroid-stimulating hormone 2.860 mIU/L (normal 0.4–5.5)
  • Red blood cell folate 539 ng/mL (normal 257–800)
  • Vitamin B12 321 pg/mL (normal 221–700)
  • Iron/total iron-binding capacity 21/445 μg/dL (normal 30–140, 210–415)
  • Ferritin 5 ng/mL (normal 9–150).

DIFFERENTIAL DIAGNOSIS

1. Which of the following is the most likely cause of her diarrhea?

  • Thyroid disease
  • Functional bowel disease
  • Gluten-sensitive enteropathy (celiac disease)

Given her constellation of symptoms (fatigue, neuropsychiatric changes, iron deficiency anemia, and diarrhea), celiac disease is the most likely diagnosis. Hyperthyroidism can cause diarrhea, but this is unlikely since her thyroid tests are normal. Functional bowel disease is a diagnosis of exclusion and usually has a more chronic, fluctuating course.

CELIAC DISEASE HAS VARIOUS PRESENTATIONS

Celiac disease has various presentations and therefore has been classified into several types1,2:

Classic disease is dominated by symptoms of malabsorption. The diagnosis is established by serologic testing, findings of villous atrophy on biopsy, and improvement of symptoms on a gluten-free diet. However, the presentation of celiac disease has changed, and now atypical presentations are more common in adults (see below). The reason for the change in presentation is not known, but some have hypothesized that it is related to an increase in breast-feeding and the later introduction of cereals into infants’ diets.

Celiac disease with atypical symptoms is characterized by extraintestinal manifestations with few or no gastrointestinal (GI) symptoms. Patients may present with iron-deficiency anemia; osteoporosis or vitamin D deficiency; arthritis; neurologic symptoms such as ataxia, headaches, or depression or anxiety; myocarditis; infertility; or elevated aminotransferase levels. As in classic celiac disease, the diagnosis is established with serologic testing, findings of villous atrophy on biopsy, and improvement of symptoms on a gluten-free diet.

Latent disease includes cases in patients with positive serologic tests but no villous atrophy on biopsy. These patients have no symptoms but may develop symptoms or histologic changes later.

Silent disease refers to cases in patients who have no symptoms but have a positive serologic test and villous atrophy on biopsy. These cases are usually detected via screening of people at high risk, ie, relatives of patients with celiac disease.

It is important that clinicians be aware of the various symptoms and presentations of celiac disease in order to make the diagnosis.

 

 

CONFIRMING CELIAC DISEASE

2. Which of the following is used to test for celiac disease?

  • Immunoglobulin G (IgG) and immunoglobulin A (IgA) antigliadin antibody testing
  • IgA antiendomysial antibody and IgA antitransglutaminase antibody testing
  • HLA DQ2/DQ8 testing

The sensitivity of antigliadin antibody testing is only about 70% to 85%, and its specificity is about 70% to 90%. Better serologic tests are those for IgA antiendomysial and antitransglutaminase antibodies, which have sensitivities greater than 90% and specificities greater than 95%.3 HLA DQ2/DQ8 testing has a high sensitivity (> 90%–95%), but because about 30% of the general population also carry these markers, the specificity of this test is not ideal. This test is best used for its negative predictive value—ie, to rule out the diagnosis of celiac disease.

Of note: 1% to 2% of patients with celiac disease have a deficiency of IgA.4 Therefore, if the clinical suspicion for celiac disease is high but the IgA antibody tests are negative or equivocal, IgG antitransglutaminase and IgG antiendomysial antibody tests can help establish the diagnosis. HLA testing in this situation can also help rule out the diagnosis.

CONFIRMING CELIAC DISEASE—CONTINUED

3. What test should be performed next in this patient?

  • Upper GI series with small-bowel follow-through
  • Esophagogastroduodenoscopy with biopsies
  • Small-bowel barium study
  • Video capsule endoscopy

Today, the presumptive diagnosis of celiac disease requires positive serologic testing and biopsy results. Esophagogastroduodenoscopy with biopsies should be ordered. Upper GI series and barium studies do not provide a tissue diagnosis. Barium studies and other radiologic tests can be considered if a patient does not have the expected response to a strict gluten-free diet or if one suspects complications of celiac disease, such as GI lymphoma.

Video capsule endoscopy is an emerging tool for diagnosing celiac disease, as suggested in several trials.5 Some findings seen on video capsule endoscopy in patients with celiac disease include mosaicism, nodularity, visible vessels, and loss of mucosal folds. However, the role of this test continues to be investigated, and biopsy is still required to confirm the diagnosis.

Figure 1. A low-power view (top left) and high-power view (top right) of the normal villous architecture of the small intestine. The high-power view shows the enterocytes and interspersed goblet cells (arrows). Bottom left, a sample from a patient with celiac disease shows severe villous blunting in this low-power view. Bottom right, a higher-power view shows loss of columnar epithelial shape, mucin depletion, and an increased nuclear-to-cytoplasmic ratio within the epithelial layer. Within the normal epithelial layer, there is an increased ratio of lymphocytes to epithelial cells. The normal ratio of lymphocytes to epithelial cells is 1 to 5–10. In this specimen the ratio is approximately 3 to 5 lymphocytes to 5 epithelial cells. The lamina propria also shows an increased number of plasma cells. This constellation of microscopic findings is characteristic of gluten-sensitive enteropathy.
A definitive diagnosis is confirmed when symptoms resolve with a gluten-free diet, and repeat biopsies (3–9 months after dietary changes) show histologic improvement, although the need for repeat biopsy is controversial. A hallmark histologic finding is villous atrophy (Figure 1). However, villous atrophy may be patchy, and it is recommended that multiple biopsy specimens be taken from the duodenal mucosa to increase the diagnostic yield.

WHO SHOULD BE TESTED FOR CELIAC DISEASE?

The reported prevalence of symptomatic celiac disease is about 1 in 1,000 live births in populations of northern European ancestry, ranging from 1 in 250 (in Sweden) to 1 in 4,000 (in Denmark).6 The prevalence appears to be higher in women than in men.7

In a large US study, the prevalence of celiac disease was 1 in 22 in first-degree relatives of celiac patients, 1 in 39 in second-degree relatives, 1 in 56 in patients with either GI symptoms or a condition associated with celiac disease, and 1 in 133 in groups not at risk.8 Another study found that the prevalence of antiendomysial antibodies in US blood donors was as high as 1 in 2,502.

Given that patients with celiac disease may not present with classic symptoms, it has been suggested that the following groups of patients be tested for it1:

  • Patients with GI symptoms such as chronic diarrhea, malabsorption, weight loss, or abdominal symptoms
  • Patients without diarrhea but with other unexplained signs or symptoms that could be due to celiac disease, such as iron-deficiency anemia, elevated aminotransferase levels, short stature, delayed puberty, or infertility
  • Symptomatic patients at high risk for celiac disease. Risk factors include type 1 diabetes or other autoimmune endocrinopathies, first- and second-degree relatives of people with celiac disease, and patients with Turner, Down, or Williams syndromes.

Screening of the general population is not recommended, even in populations at high risk (eg, white people of northern European ancestry).

 

 

WHAT CAN CELIAC PATIENTS EAT?

4. Patients with celiac disease should avoid eating which of the following?

  • Wheat
  • Barley
  • Rye
  • Oats

Patients with celiac disease should follow a gluten-free diet and should initially eliminate all of these substances.

Some recent studies have suggested that pure oat powder can be tolerated without disease recurrence, although the long-term safety of oat consumption in patients with celiac disease is uncertain.9 It may be reasonable for patients to reintroduce oats when the disease is under control, especially since uncontaminated oats can be obtained from reliable retail or wholesale stores. The definitive diagnosis of celiac disease requires clinical suspicion, serologic tests, biopsy, and documented clinical and histologic improvement after a gluten-free diet is started.

All patients with celiac disease should receive dietary counseling and referral to a nutritionist who is experienced in the treatment of this disease. Because of the significant lifestyle and dietary changes involved in treating this disease, many patients may also benefit from participating in a celiac support group.

COMPLICATIONS OF CELIAC DISEASE

5. What are the complications of untreated celiac disease?

  • Anemia
  • Osteoporosis
  • Intestinal lymphoma
  • Infertility
  • Neuropsychiatric symptoms
  • Rash

All of the above are complications of untreated celiac disease and are often clinical features at presentation. Patients with celiac disease should be tested for anemia and nutritional deficiencies, including iron, folate, calcium, and vitamin D deficiency.

All patients should also undergo dual-energy x-ray absorptiometric scanning. Bone loss is thought to be related to vitamin D deficiency and secondary hyperparathyroidism, and may be partially reversed with a gluten-free diet.

Celiac disease is associated with hyposplenism, so pneumococcal vaccination should be considered. Celiac disease is also frequently associated with the rash of dermatitis herpetiformis, and diagnosis of this rash should prompt an evaluation for celiac disease.

Other associated conditions include Down syndrome, selective IgA deficiency, and other autoimmune diseases such as type 1 diabetes, thyroid disease, and liver disease.

WHAT HAPPENED TO OUR PATIENT?

Our patient tested positive for antiendomysial and antitransglutaminase antibodies and underwent small-bowel biopsy, which confirmed the diagnosis of celiac disease. She was started on a gluten-free diet, and within 2 weeks she noted an improvement in her symptoms of fatigue, GI upset, mood disorders, and difficulty with concentration. She met with a nutritionist who specializes in celiac disease and joined a celiac support group.

However, about 2 months later, her symptoms recurred. She again met with her nutritionist, who confirmed that she was adhering to a gluten-free and lactose-free diet. Even so, when she was tested again for antitransglutaminase antibodies, the titer was elevated. Stool cultures were obtained and were negative. She was started on a course of prednisone, and her symptoms resolved.

WHAT IF PATIENTS DO NOT RESPOND TO TREATMENT?

The most common cause of recurrent symptoms or nonresponse to treatment is noncompliance with the gluten-free diet or inadvertent ingestion of gluten. Patients who do not respond to treatment or who have a period of response but then relapse should be referred back to a nutritionist who specializes in celiac disease.

If a patient continues to have symptoms despite strict adherence to a gluten-free diet, other disorders should be considered, such as concomitant lactose intolerance, small-bowel bacterial overgrowth, pancreatic insufficiency, or irritable bowel syndrome. If these conditions are ruled out, patients can be considered for treatment with prednisone or other immunosuppressive agents. Patients with refractory symptoms are at higher risk of more severe complications of celiac disease, such as intestinal lymphoma, intestinal strictures, and collagenous colitis.

TAKE-HOME POINTS

  • Celiac disease classically presents with symptoms of malabsorption, but nonclassic presentations are much more common.
  • Celiac disease should be tested for in patients with or without symptoms of mal-absorption and other associated signs or symptoms including unexplained iron-deficiency anemia, infertility, short stature, delayed puberty, or elevated transaminases. Testing should be considered for symptomatic patients with type 1 diabetes or other autoimmune endocrinopathies, first- and second-degree relatives of patients with known disease, and those with certain chromosomal abnormalities.
  • Heightened physician awareness is important in the diagnosis of celiac disease.
  • Diagnosis depends on serologic testing, biopsy, and clinical improvement on a gluten-free diet.
  • Treatment should consist of education about the disease, consultation with a nutritionist experienced in celiac disease, and lifelong adherence to a gluten-free diet. Referral to a celiac support group should be considered.
  • Long-term follow-up should include heightened vigilance and awareness of the complications of celiac disease such as osteoporosis, vitamin D deficiency and other nutritional deficiencies, increased risk of malignancy, association with low birth-weight infants and preterm labor, and occurrence of autoimmune disorders.

Acknowledgments: I would like to extend a special thank you to Dr. Walter Henricks, Director, Center for Pathology Informatics, Pathology and Laboratory Medicine, Cleveland Clinic, for providing biopsy slides and interpretation. I would also like to extend thanks to Dr. Derek Abbott, Department of Pathology, Case Western University Hospitals, for his helpful criticisms.

A 22-year-old woman presents to the clinic for evaluation of fatigue. She has not felt well for the past few years. Her current symptoms include generalized fatigue and diarrhea, characterized as two to three semi-formed, nonbloody bowel movements each day and occasional episodes of watery diarrhea. Her bowel movements are usually precipitated by meals. She consumes a regular diet and has not recognized any intolerance to any particular foods. She denies having any abdominal pain, nausea, vomiting, recent travel, joint pain, rash, or change in the texture of her hair. She has been seen by several internists in her hometown, who have not provided her with a specific diagnosis.

Her medical history is significant for anemia, anxiety, and depression. Menarche occurred at age 16. Her menstrual cycle has been regular, with bleeding noted to be only modest. Her medications include oral contraceptive pills. She has not had previous surgeries.

On examination, she appears well. She is afebrile, weighs 128 lbs, and is 63 inches tall. The physical examination is normal, including a rectal examination and fecal occult blood testing.

Routine laboratory tests are performed. Results:

  • White blood cell count 3.88 × 109/L (normal 4.0–11)
  • Hemoglobin 10.4 g/dL (normal 12–16)
  • Hematocrit 34% (normal 37%–47%)
  • Mean corpuscular volume 80.2 fL (normal 80–100)
  • Mean corpuscular hemoglobin 24.5 pG (normal 27–34)
  • Platelet count 365 × 109/L (normal 150–400)
  • Sodium 141 mmol/L (normal 132–148)
  • Potassium 4.2 mmol/L (normal 3.5–5.0)
  • Chloride 107 mmol/L (normal 98–110)
  • Alanine aminotransferase 22 U/L (normal 0–45)
  • Glucose 66 mg/dL (normal 65–100)
  • Blood urea nitrogen 6 mg/dL (normal 8–25)
  • Creatinine 0.6 mg/dL (normal 0.7–1.4)
  • Thyroid-stimulating hormone 2.860 mIU/L (normal 0.4–5.5)
  • Red blood cell folate 539 ng/mL (normal 257–800)
  • Vitamin B12 321 pg/mL (normal 221–700)
  • Iron/total iron-binding capacity 21/445 μg/dL (normal 30–140, 210–415)
  • Ferritin 5 ng/mL (normal 9–150).

DIFFERENTIAL DIAGNOSIS

1. Which of the following is the most likely cause of her diarrhea?

  • Thyroid disease
  • Functional bowel disease
  • Gluten-sensitive enteropathy (celiac disease)

Given her constellation of symptoms (fatigue, neuropsychiatric changes, iron deficiency anemia, and diarrhea), celiac disease is the most likely diagnosis. Hyperthyroidism can cause diarrhea, but this is unlikely since her thyroid tests are normal. Functional bowel disease is a diagnosis of exclusion and usually has a more chronic, fluctuating course.

CELIAC DISEASE HAS VARIOUS PRESENTATIONS

Celiac disease has various presentations and therefore has been classified into several types1,2:

Classic disease is dominated by symptoms of malabsorption. The diagnosis is established by serologic testing, findings of villous atrophy on biopsy, and improvement of symptoms on a gluten-free diet. However, the presentation of celiac disease has changed, and now atypical presentations are more common in adults (see below). The reason for the change in presentation is not known, but some have hypothesized that it is related to an increase in breast-feeding and the later introduction of cereals into infants’ diets.

Celiac disease with atypical symptoms is characterized by extraintestinal manifestations with few or no gastrointestinal (GI) symptoms. Patients may present with iron-deficiency anemia; osteoporosis or vitamin D deficiency; arthritis; neurologic symptoms such as ataxia, headaches, or depression or anxiety; myocarditis; infertility; or elevated aminotransferase levels. As in classic celiac disease, the diagnosis is established with serologic testing, findings of villous atrophy on biopsy, and improvement of symptoms on a gluten-free diet.

Latent disease includes cases in patients with positive serologic tests but no villous atrophy on biopsy. These patients have no symptoms but may develop symptoms or histologic changes later.

Silent disease refers to cases in patients who have no symptoms but have a positive serologic test and villous atrophy on biopsy. These cases are usually detected via screening of people at high risk, ie, relatives of patients with celiac disease.

It is important that clinicians be aware of the various symptoms and presentations of celiac disease in order to make the diagnosis.

 

 

CONFIRMING CELIAC DISEASE

2. Which of the following is used to test for celiac disease?

  • Immunoglobulin G (IgG) and immunoglobulin A (IgA) antigliadin antibody testing
  • IgA antiendomysial antibody and IgA antitransglutaminase antibody testing
  • HLA DQ2/DQ8 testing

The sensitivity of antigliadin antibody testing is only about 70% to 85%, and its specificity is about 70% to 90%. Better serologic tests are those for IgA antiendomysial and antitransglutaminase antibodies, which have sensitivities greater than 90% and specificities greater than 95%.3 HLA DQ2/DQ8 testing has a high sensitivity (> 90%–95%), but because about 30% of the general population also carry these markers, the specificity of this test is not ideal. This test is best used for its negative predictive value—ie, to rule out the diagnosis of celiac disease.

Of note: 1% to 2% of patients with celiac disease have a deficiency of IgA.4 Therefore, if the clinical suspicion for celiac disease is high but the IgA antibody tests are negative or equivocal, IgG antitransglutaminase and IgG antiendomysial antibody tests can help establish the diagnosis. HLA testing in this situation can also help rule out the diagnosis.

CONFIRMING CELIAC DISEASE—CONTINUED

3. What test should be performed next in this patient?

  • Upper GI series with small-bowel follow-through
  • Esophagogastroduodenoscopy with biopsies
  • Small-bowel barium study
  • Video capsule endoscopy

Today, the presumptive diagnosis of celiac disease requires positive serologic testing and biopsy results. Esophagogastroduodenoscopy with biopsies should be ordered. Upper GI series and barium studies do not provide a tissue diagnosis. Barium studies and other radiologic tests can be considered if a patient does not have the expected response to a strict gluten-free diet or if one suspects complications of celiac disease, such as GI lymphoma.

Video capsule endoscopy is an emerging tool for diagnosing celiac disease, as suggested in several trials.5 Some findings seen on video capsule endoscopy in patients with celiac disease include mosaicism, nodularity, visible vessels, and loss of mucosal folds. However, the role of this test continues to be investigated, and biopsy is still required to confirm the diagnosis.

Figure 1. A low-power view (top left) and high-power view (top right) of the normal villous architecture of the small intestine. The high-power view shows the enterocytes and interspersed goblet cells (arrows). Bottom left, a sample from a patient with celiac disease shows severe villous blunting in this low-power view. Bottom right, a higher-power view shows loss of columnar epithelial shape, mucin depletion, and an increased nuclear-to-cytoplasmic ratio within the epithelial layer. Within the normal epithelial layer, there is an increased ratio of lymphocytes to epithelial cells. The normal ratio of lymphocytes to epithelial cells is 1 to 5–10. In this specimen the ratio is approximately 3 to 5 lymphocytes to 5 epithelial cells. The lamina propria also shows an increased number of plasma cells. This constellation of microscopic findings is characteristic of gluten-sensitive enteropathy.
A definitive diagnosis is confirmed when symptoms resolve with a gluten-free diet, and repeat biopsies (3–9 months after dietary changes) show histologic improvement, although the need for repeat biopsy is controversial. A hallmark histologic finding is villous atrophy (Figure 1). However, villous atrophy may be patchy, and it is recommended that multiple biopsy specimens be taken from the duodenal mucosa to increase the diagnostic yield.

WHO SHOULD BE TESTED FOR CELIAC DISEASE?

The reported prevalence of symptomatic celiac disease is about 1 in 1,000 live births in populations of northern European ancestry, ranging from 1 in 250 (in Sweden) to 1 in 4,000 (in Denmark).6 The prevalence appears to be higher in women than in men.7

In a large US study, the prevalence of celiac disease was 1 in 22 in first-degree relatives of celiac patients, 1 in 39 in second-degree relatives, 1 in 56 in patients with either GI symptoms or a condition associated with celiac disease, and 1 in 133 in groups not at risk.8 Another study found that the prevalence of antiendomysial antibodies in US blood donors was as high as 1 in 2,502.

Given that patients with celiac disease may not present with classic symptoms, it has been suggested that the following groups of patients be tested for it1:

  • Patients with GI symptoms such as chronic diarrhea, malabsorption, weight loss, or abdominal symptoms
  • Patients without diarrhea but with other unexplained signs or symptoms that could be due to celiac disease, such as iron-deficiency anemia, elevated aminotransferase levels, short stature, delayed puberty, or infertility
  • Symptomatic patients at high risk for celiac disease. Risk factors include type 1 diabetes or other autoimmune endocrinopathies, first- and second-degree relatives of people with celiac disease, and patients with Turner, Down, or Williams syndromes.

Screening of the general population is not recommended, even in populations at high risk (eg, white people of northern European ancestry).

 

 

WHAT CAN CELIAC PATIENTS EAT?

4. Patients with celiac disease should avoid eating which of the following?

  • Wheat
  • Barley
  • Rye
  • Oats

Patients with celiac disease should follow a gluten-free diet and should initially eliminate all of these substances.

Some recent studies have suggested that pure oat powder can be tolerated without disease recurrence, although the long-term safety of oat consumption in patients with celiac disease is uncertain.9 It may be reasonable for patients to reintroduce oats when the disease is under control, especially since uncontaminated oats can be obtained from reliable retail or wholesale stores. The definitive diagnosis of celiac disease requires clinical suspicion, serologic tests, biopsy, and documented clinical and histologic improvement after a gluten-free diet is started.

All patients with celiac disease should receive dietary counseling and referral to a nutritionist who is experienced in the treatment of this disease. Because of the significant lifestyle and dietary changes involved in treating this disease, many patients may also benefit from participating in a celiac support group.

COMPLICATIONS OF CELIAC DISEASE

5. What are the complications of untreated celiac disease?

  • Anemia
  • Osteoporosis
  • Intestinal lymphoma
  • Infertility
  • Neuropsychiatric symptoms
  • Rash

All of the above are complications of untreated celiac disease and are often clinical features at presentation. Patients with celiac disease should be tested for anemia and nutritional deficiencies, including iron, folate, calcium, and vitamin D deficiency.

All patients should also undergo dual-energy x-ray absorptiometric scanning. Bone loss is thought to be related to vitamin D deficiency and secondary hyperparathyroidism, and may be partially reversed with a gluten-free diet.

Celiac disease is associated with hyposplenism, so pneumococcal vaccination should be considered. Celiac disease is also frequently associated with the rash of dermatitis herpetiformis, and diagnosis of this rash should prompt an evaluation for celiac disease.

Other associated conditions include Down syndrome, selective IgA deficiency, and other autoimmune diseases such as type 1 diabetes, thyroid disease, and liver disease.

WHAT HAPPENED TO OUR PATIENT?

Our patient tested positive for antiendomysial and antitransglutaminase antibodies and underwent small-bowel biopsy, which confirmed the diagnosis of celiac disease. She was started on a gluten-free diet, and within 2 weeks she noted an improvement in her symptoms of fatigue, GI upset, mood disorders, and difficulty with concentration. She met with a nutritionist who specializes in celiac disease and joined a celiac support group.

However, about 2 months later, her symptoms recurred. She again met with her nutritionist, who confirmed that she was adhering to a gluten-free and lactose-free diet. Even so, when she was tested again for antitransglutaminase antibodies, the titer was elevated. Stool cultures were obtained and were negative. She was started on a course of prednisone, and her symptoms resolved.

WHAT IF PATIENTS DO NOT RESPOND TO TREATMENT?

The most common cause of recurrent symptoms or nonresponse to treatment is noncompliance with the gluten-free diet or inadvertent ingestion of gluten. Patients who do not respond to treatment or who have a period of response but then relapse should be referred back to a nutritionist who specializes in celiac disease.

If a patient continues to have symptoms despite strict adherence to a gluten-free diet, other disorders should be considered, such as concomitant lactose intolerance, small-bowel bacterial overgrowth, pancreatic insufficiency, or irritable bowel syndrome. If these conditions are ruled out, patients can be considered for treatment with prednisone or other immunosuppressive agents. Patients with refractory symptoms are at higher risk of more severe complications of celiac disease, such as intestinal lymphoma, intestinal strictures, and collagenous colitis.

TAKE-HOME POINTS

  • Celiac disease classically presents with symptoms of malabsorption, but nonclassic presentations are much more common.
  • Celiac disease should be tested for in patients with or without symptoms of mal-absorption and other associated signs or symptoms including unexplained iron-deficiency anemia, infertility, short stature, delayed puberty, or elevated transaminases. Testing should be considered for symptomatic patients with type 1 diabetes or other autoimmune endocrinopathies, first- and second-degree relatives of patients with known disease, and those with certain chromosomal abnormalities.
  • Heightened physician awareness is important in the diagnosis of celiac disease.
  • Diagnosis depends on serologic testing, biopsy, and clinical improvement on a gluten-free diet.
  • Treatment should consist of education about the disease, consultation with a nutritionist experienced in celiac disease, and lifelong adherence to a gluten-free diet. Referral to a celiac support group should be considered.
  • Long-term follow-up should include heightened vigilance and awareness of the complications of celiac disease such as osteoporosis, vitamin D deficiency and other nutritional deficiencies, increased risk of malignancy, association with low birth-weight infants and preterm labor, and occurrence of autoimmune disorders.

Acknowledgments: I would like to extend a special thank you to Dr. Walter Henricks, Director, Center for Pathology Informatics, Pathology and Laboratory Medicine, Cleveland Clinic, for providing biopsy slides and interpretation. I would also like to extend thanks to Dr. Derek Abbott, Department of Pathology, Case Western University Hospitals, for his helpful criticisms.

References
  1. National Institutes of Health. NIH Consensus Development Conference on Celiac Disease, 2004 Accessed 1/29/2008. http://consensus.nih.gov/2004/2004CeliacDisease118html.htm.
  2. Rostom A, Murray JA, Kagnoff MF. American Gastroenterological Association (AGA) Institute technical review on the diagnosis and management of celiac disease. Gastroenterology 2006; 131:19812002.
  3. Hellekson K. AHRQ releases practice guidelines for celiac disease screening. Am Fam Phys 2005; 71:13.
  4. Cataldo F, Marino V, Bottaro G, Greco P, Ventura A. Celiac disease and selective immunoglobulin A deficiency. J Pediatr 1997; 131:306308.
  5. Kesari A, Bobba RK, Arsura EL. Video capsule endoscopy and celiac disease. Gastrointest Endosc 2005; 62:796797.
  6. Branski D, Fasano A, Troncone R. Latest developments in the pathogenesis and treatment of celiac disease. J Pediatr 2006; 149:295300.
  7. Rampertab SD, Pooran N, Brar P, Singh P, Green PH. Trends in the presentation of celiac disease. Am J Med 2006; 119 4:355.e9e14.
  8. Fasano A, Berti I, Gerarduzzi T, et al. Prevalence of celiac disease in at-risk and not-at-risk groups in the United States: a large multicenter study. Arch Intern Med 2003; 163:286292.
  9. Janatuinen EK, Pikkarainen PH, Kemppainen TA, et al. A comparison of diets with and without oats in adults with celiac disease. N Engl J Med 1995; 333:10331037.
References
  1. National Institutes of Health. NIH Consensus Development Conference on Celiac Disease, 2004 Accessed 1/29/2008. http://consensus.nih.gov/2004/2004CeliacDisease118html.htm.
  2. Rostom A, Murray JA, Kagnoff MF. American Gastroenterological Association (AGA) Institute technical review on the diagnosis and management of celiac disease. Gastroenterology 2006; 131:19812002.
  3. Hellekson K. AHRQ releases practice guidelines for celiac disease screening. Am Fam Phys 2005; 71:13.
  4. Cataldo F, Marino V, Bottaro G, Greco P, Ventura A. Celiac disease and selective immunoglobulin A deficiency. J Pediatr 1997; 131:306308.
  5. Kesari A, Bobba RK, Arsura EL. Video capsule endoscopy and celiac disease. Gastrointest Endosc 2005; 62:796797.
  6. Branski D, Fasano A, Troncone R. Latest developments in the pathogenesis and treatment of celiac disease. J Pediatr 2006; 149:295300.
  7. Rampertab SD, Pooran N, Brar P, Singh P, Green PH. Trends in the presentation of celiac disease. Am J Med 2006; 119 4:355.e9e14.
  8. Fasano A, Berti I, Gerarduzzi T, et al. Prevalence of celiac disease in at-risk and not-at-risk groups in the United States: a large multicenter study. Arch Intern Med 2003; 163:286292.
  9. Janatuinen EK, Pikkarainen PH, Kemppainen TA, et al. A comparison of diets with and without oats in adults with celiac disease. N Engl J Med 1995; 333:10331037.
Issue
Cleveland Clinic Journal of Medicine - 75(3)
Issue
Cleveland Clinic Journal of Medicine - 75(3)
Page Number
215-219
Page Number
215-219
Publications
Publications
Topics
Article Type
Display Headline
A young woman with fatigue
Display Headline
A young woman with fatigue
Sections
Disallow All Ads
Alternative CME
Use ProPublica
Article PDF Media